You are on page 1of 129

PUP

Instructional Materials in
GEED 10053
Mathematics in the Modern World

compiled by

DMS Faculty

College of Science
Polytechnic University of the Philippines

2020
for the sole noncommercial use of the
Faculty of the Department of Mathematics and
Statistics Polytechnic University of the Philippines

2020

Conributors:

Abdul, Alsafat
Atienza, Jacky Boy
Bang-as, Pamela
Bernardino, Rhea
Cabanig, Sarah Jean
Criseno, Regine
Dilla, Perlyn Mae
Duarte, Rafael
Elizon, Katrina
Equiza, Cynthia
Hernandez, Andrew
Isaac, Emelita
Lara, Jose Alejandro Constantino
Longhas, Paul Ryan
Macatangay, Shaina Lyra
Malvar, Rolan
Nuguid, Kenneth James
Saguindan, Ian
Sta. Maria, John Patrick
Republic of the Philippines
POLYTECHNIC UNIVERSITY OF THE PHILIPPINES
COLLEGE OF SCIENCE
Department of Mathematics and Statistics

Course Title : MATHEMATICS IN THE MODERN WORLD

Course Code : GEED 10053

Course Credit : 3 units

Pre-Requisite : GENERAL MATHEMATICS, STATISTICS AND PROBABILITY (SHS) Course

Description :

The course deals with the nature of mathematics, appreciation of its practical, intellectual and
aesthetic dimensions, and application of mathematical tools in daily life. It also bridges the study of
mathematics to other domains of interest like business, finance, social sciences and arts and design.

COURSE LEARNING PLAN

Week Dates Topics and Subtopics

I. Nature of Mathematics

Mathematics in Nature
3. Mathematics for Our World
Week 1 10/5 – 10/11

Language of Mathematics

1. Propositions and Logical


Connectives 2. Sets, Operations and
Venn Diagrams
Week 2-3 10/12 – 10/18 10/19 –
10/25
Problem Solving

1. Inductive and Deductive Reasoning


2. Polya’s Guidelines for Problem
Solving 3. Mathematical Problems
Week 4-5 10/26 – 11/1 11/2 – 11/8 involving Patterns
1. Patterns and Numbers in Nature 2.
Fibonacci Sequence
II. Mathematics as a Tool: Statistics and Data

Management Week 6 11/9 – 11/15 Data Gatheing and Sampling Techniques

1. Tabular Presentations: Frequency Distributions


and Crosstabulations
2. Graphical Presentations: Graphs, Charts, Time
Series Plots

Descriptive Measures
Week 7 11/16 – 11/22 Week 8 11/23 – 11/29
1. Measures of Central Tendency
2. Measures of Dispersion or Variability

III. Special Topics

Financial Mathematics

Week 9-11 11/30– 12/20 Week 12-14 1/4– 1/24 1. Simple and Compound Interest
2. Ordinary Annuities
3. Paying Off a Debt or Loan Repayment
4. Other Applications of Financial Mathematics

Mathematics of Voting and Apportionment


1. Steps in Statistical Investigation
2. Sampling Techniques, Sample Size 1. Voting Methods
Considerations, Methods of Data Collection 2. Apportionment
3. Levels of Measurement

FINAL ASSESSMENT
Data Presentation

*Note: Financial Mathematics and Mathematics of Voting and Apportionment are required special topics for the programs
under the following colleges: Accountancy and Finance (CAF), Arts and Letters (CAL), Business Administration (CBA),
Communication (COC), Education (CoED), Human Kinetics (CHK), Political Science and Public Administration (CPSPA),
Social Sciences and Development (CSSD) and Tourism, Hospitality and Transportation Management (CTHTM).

COURSE GRADING SYSTEM

The final grade will be based on the weighted average of the student’s scores on each test assigned at
the end of each lesson. The final SIS grade equivalent will be based on the following table according to
the approved University Student Handbook.

Class Standing (CS) = (Weighted Average of all the Chapter/Unit Tests ÷ 2) + 50% Midterm and/or
Final Exam (MFE) = (Weighted Average of the Midterm and/or FinalTests ÷ 2) + 50%

Final Grade = (70% x CS) + (30% x MFE)


SIS Grade Percentage/Equivalent Description
1.00 97.00 - 100 Excellent
1.25 94.00-96.99 Excellent
1.50 91.00-93.99 Very Good
1.75 88.00-90.99 Very Good
2.00 85.00-87.99 Good
2.25 82.00-84.99 Good
2.50 79.00-81.99 Satisfactory
2.75 77.00-78.99 Satisfactory
3.00 75.00-76.99 Passing
5.00 65.00-74.99 Failure
Inc Incomplete
W Withdrawn
Final grades are rounded off to 2 decimal places.

Reference Materials:

• Smith, Karl J. The Nature of Mathematics. 12ed. Cengage Learning. 2012


• Angel, Abbott, Runde. Survey of Mathematics with Applications. 10ed. Pearson.
2016 • Lippman, David. Mathematics in Society. 2ed. 2017
• Thomas, Christopher. Schaum’s Outline of Mathematics for the Liberal Arts. McGrawHill.

2009 Prepared by: Noted by:

Kenneth James T. Nuguid/ Ian J. Saguindan Edcon B. Baccay Faculty


Members Chairperson
Department of Mathematics and Statistics Department of Mathematics and Statistics Approved by:

Dr. Lincoln A. Bautista


Dean, College of Science

Dr. Emanuel C. de Guzman


Vice President for Academic Affairs

Contents

PUP DM S
1 Mathematics in Our World . . . . . . . . . . . . . . . . . . . . . . . . . . . . . . . . . 5 1.1 Overview: What
is mathematics? . . . . . . . . . . . . . . . . . . . . . . . . . 5 1.2 Patterns and Numbers in Nature
. . . . . . . . . . . . . . . . . . . . . . . . . 5 1.3 Fibonacci Sequence . . . . . . . . . . . . . . . . . . . . . . .
. . . . . . . . . 12 1.4 Mathematics for Our World . . . . . . . . . . . . . . . . . . . . . . . . . . . . 17
2 Logic and Sets . . . . . . . . . . . . . . . . . . . . . . . . . . . . . . . . . . . . . . . 22 2.1 Propositions . . . .
. . . . . . . . . . . . . . . . . . . . . . . . . . . . . . . . 22 2.2 Compound Propositions . . . . . . . . . . . .
. . . . . . . . . . . . . . . . . . 23 2.3 Sets . . . . . . . . . . . . . . . . . . . . . . . . . . . . . . . . . . . . . . . .
28
3 Problem Solving . . . . . . . . . . . . . . . . . . . . . . . . . . . . . . . . . . . . . . 34 3.1 Inductive and
Deductive Reasoning . . . . . . . . . . . . . . . . . . . . . . . . 34 3.2 George Polya’s Guidelines
for Problem Solving . . . . . . . . . . . . . . . . . . 37
4 Statistics and Data Management . . . . . . . . . . . . . . . . . . . . . . . . . . . . . 41 4.1 Basic
Concepts . . . . . . . . . . . . . . . . . . . . . . . . . . . . . . . . . . . 41 4.2 Steps in Statistical
Investigation . . . . . . . . . . . . . . . . . . . . . . . . . . 45 4.3 Sampling and Sampling
Techniques . . . . . . . . . . . . . . . . . . . . . . . . 45 4.4 Sample Size Considerations . . . . . . .
. . . . . . . . . . . . . . . . . . . . . 46 4.5 Methods of Data Collection . . . . . . . . . . . . . . . . . . . . .
. . . . . . . 49 4.6 Levels of Measurement . . . . . . . . . . . . . . . . . . . . . . . . . . . . . . . 49 4.7
Presentation of Data . . . . . . . . . . . . . . . . . . . . . . . . . . . . . . . . 50 4.8 Measures of
Central Tendency . . . . . . . . . . . . . . . . . . . . . . . . . . . 54 4.9 Measures of Dispersion or
Variability . . . . . . . . . . . . . . . . . . . . . . . 58
5 Financial Mathematics . . . . . . . . . . . . . . . . . . . . . . . . . . . . . . . . . . . 64 5.1 Simple and
Compound Interest . . . . . . . . . . . . . . . . . . . . . . . . . . 64 5.2 Ordinary Annuities . . . . . . .
. . . . . . . . . . . . . . . . . . . . . . . . . . 69 5.3 Loan Repayment or Paying Off a Debt . . . . . . .
. . . . . . . . . . . . . . . 72
Lesson 0 4

5.4 Other Applications of Financial Mathematics . . . . . . . . . . . . . . . . . . . 78 6 Voting


Methods and Apportionment . . . . . . . . . . . . . . . . . . . . . . . . . . . . 83 6.1 Voting Methods .
. . . . . . . . . . . . . . . . . . . . . . . . . . . . . . . . . 83 6.2 Apportionment . . . . . . . . . . . . . . . . . . . .
. . . . . . . . . . . . . . . 93
PUP DM S
All Rights Reserved. 2020 Abdul, Atienza, et. al.

Lesson 1 5 Lesson 1: Mathematics in Our World

Learning Outcomes

At the end of the lesson, the students are able to:

1. identify patterns in nature in the world;

PUP DM S
2. articulate the importance of mathematics in one’s life;

3. argue about the nature of mathematics, what it is how it is expressed, represented

and used; 4. enumerate and discuss the role of mathematics in various disciplines;

5. express appreciation for mathematics as a human endeavor.


1.1 Overview: What is mathematics?
Mathematics can be defined in many ways. For some people, Mathematics is just the study of
numbers. For others, it is a set of problem-solving tools, a language, a process of thinking, and
a study of patterns among others. Whatever point of view is taken, there is no denying the
reality that mathematics is everywhere. Individuals from around the world use math in their
daily lives. Mathematics has various applications in the world. However, Mathematics is not
only concerned with everyday problems, but also with using imagination, intuition and
reasoning to find new ideas and to solve puzzling problems. Math ematics is a branch of
science, which deals with numbers and their operations. It involves calculation, computation,
solving of problems etc. Its dictionary meaning states that, ‘Mathematics is the science of
numbers and space’ or ‘Mathematics is the science of measurement, quantity and magnitude.’
It is exact, precise, systematic and a logical subject.

Mathematics helps us to organize and systemize our ideas about patterns; in so doing, not
only can we admire and enjoy these patterns, we can also use them to infer some of the
underlying principles that govern the world of nature.

In this lesson, attention will be focused on the nature of mathematics, patterns and numbers in
nature and the world and the uses of mathematics.

1.2 Patterns and Numbers in Nature


What are patterns anyway? We usually think of it as anything that repeats again and again. A
pattern is an arrangement which helps observers anticipate what they might see or what
happens next. A pattern also shows what may have come before. A pattern organizes
information so that it becomes more useful.

All Rights Reserved. 2020 Abdul, Atienza, et. al.


Lesson 1 6

The human mind is programmed to make sense of data or to bring order where there is
disorder. It seeks to discover relationships and connections between seemingly unrelated bits
of information. In doing so, it sees patterns.

According to the National Council of Teachers of Mathematics


(1991) defines the nature of mathematics as follows: Mathe
PUP DM S
matics is a study of patterns and relationship, a way of thinking,
an art, a language, and a tool. It is about patterns and rela
tionships. Numbers are just a way to express those patterns
and relationships. patterns

Patterns are everywhere. They are deeply embedded all around


us. You can observe patterns- things like colors, shapes, ac
tions, line or curves of building, pathways or even in the gro
cery store where boxes of various items are lined up. Number
patterns such as 2,4,6,8 and 5,10,15,20 are among the first
patterns encountered in younger years.

As we advance, we encounter more patterns and discover that number patterns are not
restricted to a few types. They could be ascending, descending, multiples of a certain number.
We learned patterns through the concept of functions and sequences like arithmetic and
geometric sequences. Number patterns, logic patterns, geometric patterns and word patterns
are examples of the various patterns we learned in school. However, patterns are not limited to
these types. One can observe patterns in nature, art, architecture, human behavior, anywhere.
On this section, we will discuss the different patterns in nature, arts and architecture.

Patterns in nature are visible regularities of form found in the natural world. These patterns
recur in different contexts and can sometimes be modeled mathematically. Natural patterns
include symmetries, fractals, spirals, meanders, waves, foams, tessellations, cracks, and
stripes. Studying patterns allows one to watch, guess, create, and discover. The present
mathematics is considerably more than arithmetic, algebra, and geometry. The method of
doing it has advanced from simply performing computations or derivations into observing
patterns, testing guesses, and evaluating results.
Let us focus on the different types of symmetric patterns, analyze and observe the similarities
as well as the differences and give examples of these types of patterns as seen in nature, arts,
architecture and mathematics.

All Rights Reserved. 2020 Abdul, Atienza, et. al.


Lesson 1 7

Symmetry

When we think of patterns, we usually think of it as something that repeats again and again.
The math of symmetry can describe what this repetition may look like and as well as why some
objects seem more orderly and organized than others. That is why we can say symmetry is the
fundamental “language” of patterns.

PUP DM S
What is symmetry? Can you give examples of objects that are symmetric? Why do you
consider them symmetric? Is it because of balanced proportions? Or is it because you can
rotate, translate or reflect and they still look the same?

Symmetry can be found everywhere. It can be seen from different viewpoints namely; nature,
the arts and architecture, mathematics; especially geometry and science. Symmetry occurs
when there is congruence in dimensions, due proportions and arrangement. It provides a
sense of harmony and balance. In fact, symmetry is one of the foremost predominant themes
in arts, design and architecture all over the world and throughout human history. Mathematical
symmetry can also be explained as the passage of time, a spatial relationship and an aesthetic
element found within abstract objects, theoretic models, language, music and even knowledge
itself.

Reflection or Bilateral Symmetry

Bilateral or reflection symmetry is the simplest kind of symmetry. It is one of the most common
kinds of symmetry that we see in the natural world. It can also be called mirror symmetry
because an object with this symmetry looks unchanged if a mirror passes through its middle. In
other words, the objects have a left side and a right side that are mirror images of each other. If
a shape can be folded in half so that one half fits exactly on top of the other, then we say that
the shapes are symmetric. The fold is called a line of symmetry because it divides the shape
into two equal parts. Bilateral-symmetric objects have at least one line or axis of symmetry. The
lines of symmetry may be in any direction.

All Rights Reserved. 2020 Abdul, Atienza, et. al.


Lesson 1 8

PUP
DMS
Images Exhibiting Bilateral Symmetry

Radial Symmetry

Radial symmetry is rotational symmetry around a fixed point known as the center. Images with
more than one lines of symmetry meeting at a common point exhibits a radial symmetry. An
equilateral triangle and circles are examples. You can cut along three different axes on the
equilateral triangle while a circle can be cut along an infinite number of axes. Consider the
photo below. It has rotational symmetry. How many lines of symmetry are possible?

Radial symmetry can be found both in natural and human made objects. The photos below are
examples of rotational symmetry that can be found in the world around us.

All Rights Reserved. 2020 Abdul, Atienza, et. al.


Lesson 1 9

PUP
DMS
Did you know that there are other classifications of symmetric patterns. Patterns in the plane
are usually divided into three groups. These are rosette patterns (those that repeat in no
direction), frieze patterns ( those that repeat in exactly one direction) and wallpaper patterns
(those that repeat in more than one direction). Let us define, discuss and identify examples of
these patterns from nature and the arts. Included in the discussion is what we call tessellations
which completely cover a plane without gaps or overlaps, like wallpaper patterns.

Rosette Patterns

Rosette patterns consist of taking motif or an element and rotating and/or reflecting that
element. There are two types of rosette patterns namely cyclic and dihedral. A rosette pattern
is cyclic if it only admits rotational symmetries. On the other hand, a rosette pattern is dihedral
if it admits both rotational symmetries and bilateral or reflectional symmetries. The figures
below exhibit rosette patterns. Can you identify which of them are cyclic? dihedral?

Frieze Patterns

A frieze or border pattern is a pattern in which a basic motif repeats itself over and over in one
direction. It extends to the left and right in a way that the pattern can be mapped onto itself by a

All Rights Reserved. 2020 Abdul, Atienza, et. al.


Lesson 1 10

horizontal translation. We can usually find these patterns in unique places like on the walls of
buildings, fabrics, borders of rugs and tiled floor.

Mathematicians have already classified all the different types of frieze patterns. It turns out that
there are only seven types.

1. Hop. The frieze pattern only admits a translational symmetry.


PUP DM S
2. Step. The frieze pattern only admits a translational and glide

symmetries. 3. Sidle. The frieze pattern only admits translations and

vertical reflections.

4. Spinning Hop. The frieze pattern only admits translations and 180◦rotations (half-turns).

5. Spinning Siddle. The frieze pattern only admits translations, vertical reflections, rotations,
and glide reflections.
All Rights Reserved. 2020 Abdul, Atienza, et. al.
Lesson 1 11 6. Jump. The frieze pattern only admits translations, a horizontal reflection, and

glide reflection.

7. Spinning Jump. The frieze pattern admits translations, vertical reflections, horizontal
reflections, rotations, and glide reflections.

PUP DM S
Mathematician John B. Conway invented the names of these frieze patterns.

Wallpaper Patterns

A wallpaper pattern is a pattern with translation symmetry in two directions. It is, therefore,
essentially an arrangement of friezes stacked upon one another to fill the entire plane. Any
particular wallpaper pattern is made up of a combination of the following symmetries; reflection,
rotation and glide reflection. According to Nocon (2016), in order for a plane figure to be
considered a wallpaper pattern, it must have at least the basic unit, one copy by translation,
and a copy of these two by translation in the second direction. There must be at least two rows,
each one of at least two units long.

Beautiful patterns can be created by repeating geometric and artistic motifs according to the
symmetry of the wallpaper groups, as exemplified in works by M. C. Escher and in the patterns
created by I. Bakshee

All Rights Reserved. 2020 Abdul, Atienza, et. al.


Lesson 1 12 in the Wolfram Language using Artlandia, illustrated above. There are 17 different

wallpaper patterns.

Using intricate techniques, mathematicians were able to classify every wallpaper patterns
possible. It is shown that there are only 17 distinct types of wallpaper patterns.

PUP DM S
Some Wallpaper Patterns
Tesselations

A tessellation or tiling is a repeating pattern of figures that covers a plane with no gaps or
overlaps. It is just like a wallpaper group in which patterns are created by repeating a shape to
fill the plane.

Tessellations can be created with translations, rotations, and reflections. Tessellations can be
seen in nature, arts and everyday life. Pavements, snake skin, turtle shell and a honeycomb
are just few of many examples of tessellation we see around us. A honeycomb is a perfect
example of a natural tessellation. It uses regular hexagons to form this natural mosaic around
the surface area of the hive. Since these are regular hexagons, each interior angle of each
hexagon are 120 degrees, and all the angles in one of the hexagons equal 720 degrees.

Examples of Tesselations

1.3 Fibonacci Sequence


We start with 1 and another 1. Add them, we get 2. Add 1 and 2, we get 3. Add 2 and 3, we get
5. Add 3 and 5, we get 8. If we continue repeating the process, we obtain the sequence

1; 1; 2; 3; 5; 8; 13; : : :

All Rights Reserved. 2020 Abdul, Atienza, et. al.


Lesson 1 13

which is known as the Fibonacci sequence. The Fibonacci sequence was invented by the
Italian Leonardo Pisano Bigollo (1180-1250), who is known in mathematical history by several
names: Leonardo of Pisa (Pisano means “from Pisa”) and Fibonacci (which means “son of
Bonacci”). To formally, define the Fibonacci sequence, we start by defining F1 = 1 and F2 = 1.
For n > 2, we define
Fn := Fn−1 + Fn−2:
PUP DM S
The sequence F1; F2; F3; : : : is then the Fibonacci sequence. Such a definition is called a
recursive definition because it starts by defining some initial values and defines the next term
as a function of the previous terms.

If we take the ratio of Fn to Fn−1 for n ≥ 1,


1:61904 : : : 10 55
1:61764 : : : 11 89
n Fn Fn=Fn−1 1 1 - 2
1:61818 : : : 12 144
1 1 3 2 2 4 3 1.5 5 5
1:61797 : : : 13 233
1:666 : : : 6 8 1.6 7
1:61805 : : : 14 377
13 1.625
1:618025 : : :
n Fn Fn=Fn−1 8 21
1:61538 : : : 9 34

we see that as n gets larger and larger, the ratio gets closer and closer to a value denoted by ’.
The number ’ is called as the golden ratio and can be formally defined as

’ := lim n−→∞Fn
Fn−1:

The symbol lim n−→∞means ‘the limit as n approaches infinity’ which is usually studied in a
calculus course. It can be calculated that the exact value of ’ is

’ =1 + 5
2≈ 1:6180339887 : : : :

If we denote by ’ :=1 − 5

2, we can write the nth Fibonacci number explicitly using the formula

Fn =’n − ’n

5:
This is known as the Binet Formula.

All Rights Reserved. 2020 Abdul, Atienza, et. al.


Lesson 1 14

2
3

11

PUP DM S
5

Do you see the Fibonacci Numbers? The red curve is known as the Fibonacci

Spiral. A rectangle whose side ratio (length:width) equals ’ is called a golden rectangle.

George Dvorsky (2013) emphasized that the Fibonacci sequence has captivated
mathematicians, scien tists, artists and designers for centuries. It is a sequence with many
interesting properties. Among these is its visibility in nature. Most, if not all, natureâĂŹs most
beautiful patterns contain Fibonacci numbers.

The Fibonacci numbers appear in nature in various places. These numbers are evident at the
flower head of a sunflower or daisy. Spirals are also easier to see and to count on pineapples
and pine cones. Fibonacci numbers are there on broccoli florets and flowers and on the
arrangement of leaves around stems on many plants too.
• Pinecones, Speed Heads, Vegetables and Fruits
Spiral patterns curving from left and right can be seen at the array of seeds in the center
of a sunflower. The sum of these spirals when counted will be a Fibonacci number. You
will get two consecutive Fibonacci numbers if you divide the spirals into those pointed left
and right. The seed pods on a pinecone are also arranged in a spiral pattern. Each cone
consists of a pair of spirals, each one spiraling upwards in opposing directions. Spiral
patterns can also be deciphered in cauliflower and pineapples. Fibonacci sequence
appears on these fruits and vegetables.

• Flowers and Branches


Most flowers express the Fibonacci sequence if you count the number of petals on these
flowers. For example, lilies and irises have three petals, roses and buttercups have five,
delphiniums have eight petals and so on. Some plants also exhibit the Fibonacci
sequence in their growth points, on the places where tree branches form or split. A trunk
grows until it produces a branch, resulting

All Rights Reserved. 2020 Abdul, Atienza, et. al.


Lesson 1 15

in two growth points. The main trunk then produces another branch, resulting in three
growth points and then the trunk and the first branch produce two more growth points,
bringing the total to five as illustrated on the image below.

PUP DM S
• Honeybees
The family tree of a honey bee perfectly resembles the Fibonacci sequence. A honeybee
colony consists of a queen, a few drones and lots of workers. The following image below
shows how the family tree relates.
• The Human Body
The human body has many elements that show the Fibonacci numbers and the golden
ratio. Most of your body parts follow the Fibonacci sequence and the proportions and
measurements of the human body can also be divided up in terms of the golden ratio.

• Geography, Weather and Galaxies Fibonacci numbers and the relationships between
these numbers are evident in spiral galaxies, sea wave curves and in the patterns of
stream and drainages. Weather patterns, such as hurricanes and whirlpools sometimes
closely resemble the Golden Spiral. The milky way galaxy and some other galaxies have
spiral patterns. Planets of our solar system and their orbital periods are closely related to
the golden ratio.

All Rights Reserved. 2020 Abdul, Atienza, et. al.


Lesson 1 16

PUP
DMS
The Golden Ratio and/or the Golden Spiral can also be observed in music, art, and designs.
Appearing in many architectural structures, the presence of the golden ratio provided a sense
of balance and equilibrium. Let’s take a look at a couple of examples.

• Architecture. The Great Pyramid of Giza: The Great Pyramid of Giza built around 2560 BC
is one of the earliest examples of the use of the golden ratio. The length of each side of
the base is 756 feet, and the height is 481 feet. So, we can find that the ratio of the vase
to height is 756=481 = 1:5717:

The Greek sculptor Phidias sculpted many things including the bands of sculpture that
run above the columns of the Parthenon. Other architectural structures that exhibits the
Golden ratio include the ff: Porch of Maidens, Acropolis, Athens; Chartres Cathedral; and
Le Corbussier. Can you name other structures that has the Golden Ratio?

• Arts. Mona-Lisa by Leonardo Da Vinci: It is believed that Leonardo, as a mathematician


tried to incorporate of mathematics into art. This painting seems to be made purposefully
line up with

All Rights Reserved. 2020 Abdul, Atienza, et. al.


Lesson 1 17

golden rectangle.

An Old man by Leonardo Da Vinci: Leonardo Da Vinci explored


the human body involving in the ratios of the lengths of various
body parts. He called this ratio the "divine proportion" and
featured it in many of his paintings. We also have the The
Vetruvian Man (“The Man in Action”) by Leonardo Da Vinci;
PUP DM S
Holy Family by Micahelangelo; Crucifixion by Raphael; The
sacrament of the Last Supper by Salvador Dali (1904-1989),
and many more.

1.4 Mathematics for Our World

Mathematics is everywhere; whether it is on land, sea or air, online or on the front line,
mathematics underpins every nook and cranny of modern life. Far from a quaint subject to be
forgotten upon leaving school, it is the glue that holds our world.

Roger Bacon (1214-1294), an English Franciscan friar, philosopher, scientist and scholar of the
13th century, once stated: “Neglect of mathematics works injury to all knowledge, since he who
is ignorant of it cannot know the other sciences or the things of the world.”

Math helps us understand or make sense of the world - and we use the world to understand
math. It is therefore important that we learn math contents needed to solve complex problems
in a complex world; learn the mathematical knowledge and skills we need to understand the
world and make contributions to the global community.

Applications of Mathematics in Our World

Mathematics has so many uses of applications.

• Mathematics helps organize patterns and regularities in the world;

• Mathematics helps predict the behavior of nature and many phenomena;


• Mathematics helps control nature and occurrences in the world for our own

good; • Mathematics has applications in many human endeavors.

All Rights Reserved. 2020 Abdul, Atienza, et. al.


Lesson 1 18

PUP
DMS
Mathematics helps organize patterns and regularities in the World

According to Ian Stewart (1995), we live in a universe of patterns. Human mind and culture
have de veloped a formal system of thought for recognizing, classifying and understanding
patterns. This formal system of thought is what we know now as mathematics. We use
mathematics to organize and system atize our thoughts and ideas about patterns and other
regularities in this world. The development of these new mathematical theories helped paved
the way to the thorough understanding of the different patterns in nature. Stewart (1995) also
mentioned that our newfound understanding of natural order and nature’s secret regularities is
being used to steer artificial satellites to new destinations with far less fuel than anybody had
thought possible, to help avoid wear on the wheels of locomotives and other rolling stock, to
improve the effectiveness of heart pacemakers, to manage forests and fisheries, even to make
more efficient dishwashers. But most important of all, it is giving us a deeper vision of the
universe in which we live, and of our own place in it. Yes, mathematics has indeed helped
organize patterns and consistencies in the world.

Mathematics helps predict the behavior of nature and many phenomena.

Mathematics is used to explain why the Sun set, where it went and why it returned because it
was easier to count these events in numbers than to put them into words. Based on historical
patterns, we can make forecasts or predictions to help us prepare for our daily
activities.Formulas and other mathematical meth ods became a way of using numbers to show
how things in nature happen, where and when it will happen.

All Rights Reserved. 2020 Abdul, Atienza, et. al.


Lesson 1 19

Earth scientists have relied in the past on statistical methods to forecast natural hazard events.
However, Benoit Mandelbrot, a professor of mathematical sciences at Yale University
described how he has been using fractals to find order within complex systems in nature, such
as the natural shape of a coastline. As a result of his research, earth scientists are taking
Mandelbrot’s fractal approach one step further and are measuring past events and making
probability forecasts about the size, location, and timing of future natural disasters.
Mathematics helps control nature and occurrences in the world for our own good.
PUP

DMS
Mathematical modelling and control theory can be used. By mathematical modeling we see the
inputs to events and their most likely outcomes. Knowing these inputs and seeing their
consequences and establishing their relationship defined quantitatively, we can prepare for
calamities or natural disasters, or better yet, we can probably stop them from happening.

Control theory is defined as a field of applied mathematics that is relevant to the control of
certain phys ical processes and systems. As long as human culture has existed, control has
meant some kind of power over the environment and control theory may be viewed as the
science of modifying that environment, in the physical, biological, or even social sense. Control
theory played a major role in many technological advances in the second half of the 20th
century.

Mathematics has applications in many human endeavors making it indispensable.

Mathematics existed since the beginning of time, written or unwritten. Its unwritten history is
carved in all things found in cosmos , found in the patterns created in nature, appreciated in the
juxtaposition of the heavens and the earth, contrast between darkness and light , made sense
in the harmony created not just by a well-known orchestra but even by the rain drops falling on
offshore wind-turbines. Its language, though considered by many as abstract is in fact easy to
grasp when the logic and formula that govern it are understood by the inquisitive minds of
students, bakers, chemists , carpenters and appreciated by the receptive hearts of the
musicians - drummers, guitarists, pianists and composers; dance choreographers, gymnasts
and marathon runners.

Mathematics permeates every area of man’s life , leaving every man convinced of its value. As
a tool, mathematics is indispensable. It is needed by all people in honing their logical thinking
and reasoning, in making wise financial decisions - in budgeting or making both ends meet
when financial resources are scarce. It is needed in choosing the best interior and outdoor
designs of houses , offices and business sites. It is useful in determining traveling time and
calculating the amount of fuel needed to get to the destination. It is not just needed in the
classrooms but also at home when doing the mundane baking or preparing foods for breakfast
, dinner or lunch; calculating steps when performing simple to complex acrobatic stance;
determining speed in a short distance or marathon run, preparing chemical solutions in

All Rights Reserved. 2020 Abdul, Atienza, et. al.


Lesson 1 20

a biological or chemical laboratory and the like. Indeed, its application and use are
uncountable and the list of uses it offers is unending.

As it is valuable and integral in the life of man, mathematics as a discipline that


Introduces students with the wide array of possibilities from honing problem-solving
skills to enriching aesthetic judgment.

PUP DM S
All Rights Reserved. 2020 Abdul, Atienza, et. al.

Lesson 2 21 Assessment

I. Patterns and Numbers in Nature

(1) Give five examples each of nature having reflection symmetry and radial symmetry.

PUP
DMS

(2) Compare and contrast (a) rotation and reflection; (b) translation and rotation.

(3) Which upper case letters of the English alphabet look the same after being rotated
90◦? 180◦?

(4) Classify the following frieze patterns based on Conway’s classification.

(c)
(a)

(d)

(b)
(e)

II. Fibonacci Sequence

(1) Enumerate the first twenty Fibonacci numbers.


(2) Use F40 = 63; 245; 986 and F38 = 39; 088; 169 to find the value of F39. Show your

reasoning. (3) Using the Binet’s formula, calculate F4.

III. Beyond the Walls (Performance Task)


Look for patterns Inside or outside of your house then take pictures of the patterns
explored using smart phones or digital camera. Explore, take photos, make list and
identify what patterns can be seen in nature inside your house, at the garden or park
nearby or any part of the neighborhood. Showcase your drawing skills by creating original
paintings or pictures, poster, photo collage or vlogs of the different patterns in nature,
Fibonacci, golden ratio or the like that you have encountered on your walk.

All Rights Reserved. 2020 Abdul, Atienza, et. al.

Lesson 2 22 Lesson 2: Logic and Sets

Learning Outcomes

At the end of the lesson, the students are able to

1. identify which are propositions and which are not;

PUP DM S
2. construct compound propositions using logical connectives;

3. construct truth tables for propositions;

4. test validity of arguments

2.1 Propositions
Mathematics is a language. As in any other types of language, we use sentences to
communicate thoughts and ideas. Mathematics is not an exception. We use propositions to
communicate mathematical ideas precisely.

Definition 1
A proposition is a declarative sentence that can be objectively identified as either true or
false, but not both. If a proposition is true, then its truth value is true and is denoted by T
or 1; otherwise, its truth value is false and is denoted by F or 0.

Example 1. Consider the following sentences.


(5) Is that your laptop?
(1) Douglas MacArthur arrived in the Philippines
in 1521. (6) Basketball players are handsome. (7)

(2) Are you insane? There is life in other planets. (8)



(3) 2 is an irrational number. Welcome to the Philippines!

(4) Find all x such that xe−x = 2.


Immediately, we find that sentences (2), (4), (5), and (8) are not declarative sentences, so they
are not propositions.

Sentence (1) is a proposition because Douglas MacArthur either arrived in the Philippines in
1521 or not. In fact, this proposition is false because historical records shows that Douglas
MacArthur arrived in the Philippines some time in 1900s.

All Rights Reserved. 2020 Abdul, Atienza, et. al.


Lesson 2 23

Sentence (3) is clearly a true proposition. Although statement (6) is a declarative sentence, it
cannot be considered a proposition because the meaning of the word “handsome” is subjective
in nature. Unless we could agree on an objective definition of “handsome”, then statement (6)
cannot be considered a proposition.

Finally, statement (7) is a proposition. Whether there is life or not in other planets, it doesn’t
really matter. The fact that this sentence is either true or false, and cannot be both true and
false, makes it a

PUP

DMS
proposition. For this example, we still don’t have enough evidence to claim that proposition (7)
is true yet, and we don’t have a proof that it is false either. Hence, only time will tell when can
we assign a truth value for (7), but certainly, it has a truth value.

Symbolically, we denote propositions in this lesson using lower case letters, such as p; q; r; s;
etc.

Definition 2
The negation of a proposition p is the proposition which is false when p is true; and true
when p is false. The negation of p is denoted by ¬ p.

In the English language, we can simply state the negation of a proposition p by saying “It is not
the case that p.” However, there are many ways to express negations of statements
grammatically by replacing “is/are” by “is not/are not”, etc.

Example 2. Given the statements


p : Everyone in Visayas speaks Cebuano.
q : Today is Wednesday.

The corresponding negations are

¬ p : Not everyone in Visayas speaks Cebuano.


¬ q : Today is not Wednesday.

2.2 Compound Propositions


A simple proposition is a proposition with only one subject and only one predicate. For
example, the proposition “Every cat that barks has a PhD.” is a simple proposition. The subject
of this proposition is “every cat that barks” and the predicate is “has a PhD.” In logic, we can
combine simple propositions to form compound propositions using logical connectives. Some
of the most common connectives are “or”, “and”, “but”, “unless”, etc.

All Rights Reserved. 2020 Abdul, Atienza, et. al.


Lesson 2 24

Definition 3
Let p and q be given propositions. The conjunction of p and q is the proposition “p and q”,
denoted by p ∧ q, which is true only when both p and q are true.

In other words, if one of p or q is false, then p ∧ q is false. We summarize this idea using the
following table.

PUP DM S
pqp∧q
111
100
010
000

Such a table is called a truth table for p∧q. The truth table above illustrates the different
combinations of truth values for p and q and the corresponding truth value for the conjunction.

Example 3. Given the propositions

p : 3 is odd.
q : Elephants are mammals.
r : Philippines is a first world country.

We know that p and q are true and r is false. Therefore,

p ∧ q : 3 is odd and elephants are mammals.

is true, while
p ∧ r : 3 is odd and Philippines is a first world country.

is false. For a more complicated example, the proposition

(¬ p) ∧ (¬ q) : Neither 3 is odd nor Philippines is a first world country.

is still false, since ¬ p is false.

Definition 4
Let p and q be given propositions. The disjunction of p and q is the proposition “p or q”,
denoted by p ∨ q, which is false only when both p and q are false.

In other words, if one of p or q is true (or both), then p ∨ q is true. The truth table for p ∨ q is
given below.

All Rights Reserved. 2020 Abdul, Atienza, et. al.


Lesson 2 25

pqp∨q
111
101
011
000

Example 4. Consider the statements p, q and r in the preceding example. The statement
PUP

DMS
p ∨ q : Either 3 is odd or elephants are mammals.

is true. Also,
p ∨ r : Either 3 is odd or Philippines is a first world country:

Example 5. The proposition “Either 3 is odd or there is life in other planets.” is technically true
since the component “3 is odd.” is a true proposition. Whether the proposition “There is life in
other planets.” is true or false, the disjunction is always true.

Example 6. Construct a truth table for the compound statement p ∨ (q ∧ (¬ r)).

Solution. Since each of p, q, and r may assume two distinct truth values, then there are a total
of 2 · 2 · 2 = 8 combinations, hence the truth table must contain eight rows as shown below.

p q r ¬ r q ∧ (¬ r) p ∨ (q ∧ (¬r))
111001
110111
101001
100101
011000
010111
001000
000100

Definition 5
Let p and q be propositions. The conditional statement p −→ q is the proposition “If p,
then q.” is the proposition which is false only when p is true and q is false. The converse,
inverse, and contrapositive of p −→ q are the conditional statements q −→ p, (¬ p) −→ (¬
q), and (¬ q) −→ (¬ p), respectively.

All Rights Reserved. 2020 Abdul, Atienza, et. al.


Lesson 2 26 The following is the truth table for p −→ q.

p q p −→ q
111
100
011
001

PUP DM S
In the proposition p −→ q, the proposition p is also called as the premise and q is called as the
conclusion. From the truth table, we can see that a conditional statement is trivially true when
the premise is false.

Example 7. Suppose that your mother exclaims “If you don’t wash the dishes, then you don’t
get money for a buffet.” In this conditional statement, the premise is “You don’t wash the
dishes.” and the conclusion is “you don’t get money for a buffet.” This statement can only false
only when you don’t wash the dishes but you still get money for the buffet.

Note that there are many ways to say p −→ q aside from “If p, then q.” Alternatively, we can
say “q if p” or “p implies q”, “p is sufficient for q” or “q is necessary for p.”

Example 8. Given the statements p : “ı is irrational.” and q : “3 is less than 2.”,

then p −→ q : If ı is irrational, then 3 is less than 2.

the converse of this conditional is

q −→ p : If 3 is less than 2, then ı is irrational.

the inverse is
(¬ p) −→ (¬ q) : If ı is not irrational, then 3 is not less than 2.

and the contrapositive is

(¬ q) −→ (¬ p) : If 3 is not less than 2, then ı is not irrational.

If we assume that p is true and q is false (just like how they really are in mathematics), one
verifies that both p −→ q and (¬ q) −→ (¬ p) are false, while both q −→ p and (¬ p) −→ (¬ q)
are true.

We like to emphasize that we write the negation of “ı is irrational” as “ı is not irrational” to


emphasize the fact that we actually don’t assume that the opposite of being irrational is being
rational, unless otherwise stated.

All Rights Reserved. 2020 Abdul, Atienza, et. al.


Lesson 2 27

Definition 6
Let p and q be propositions. The biconditional statement p ↔ q to be read as “p if and
only if q” is the proposition which is true only if both p and q are true or both p and q are
false.

pqp↔q
111
PUP DM S
100
010
001

Definition 7
A compound proposition is a tautology if its truth value remains true regardless of the
truth values of its component propositions. On the other hand, a compound proposition is
a contradiction if its truth value remains false regardless of the truth values of its
component propositions.

Example 9. The compound statement p ∨ (¬ p) is a tautology and the compound statement p


∧ (¬ p) is a contradiction. This can be observed by looking at the truth table below.

p ¬ p p ∨ (¬ p) p ∧ (¬ p)
1010
0110

Definition 8
Let p and q be propositions (possibly compound). We say that p logically implies q,
expressed as p =⇒ q, if the conditional statement p −→ q is a tautology. If p =⇒ q and q
=⇒ p, we say that p and q are logically equivalent and we write p ⇐⇒ q. A compound
proposition that is neither a tautology nor a contradiction is called a contingency.

Example 10. By constructing truth tables, show that p =⇒ p ∨ q and p ∧ q =⇒ p.

Solution.
p q p ∨ q p ∧ q p −→ (p ∨ q) (p ∧ q) −→ p
111111
101011
011011
000011

All Rights Reserved. 2020 Abdul, Atienza, et. al.


Lesson 2 28

In logic, the implication p =⇒ (p∨q) is called as the law of addition and the implication (p∧q)
=⇒ p is the law of simplification.

The following are some of the most common equivalences in logic.

Theorem 1
Let p; q; and r be propositions.

PUP DM S
1. p ⇐⇒ q if and only if p ←→ q is a tautology.

2. p ⇐⇒ p.

3. p ∨ q ⇐⇒ q ∨ p and p ∧ q ⇐⇒ q ∧ p. (commutative properties)

4. p ∨ (q ∨ r) ⇐⇒ (p ∨ q) ∨ r and p ∧ (q ∧ r) ⇐⇒ (p ∧ q) ∧ r. (associative properties)

5. p ∨ (q ∧ r) ⇐⇒ (p ∨ q) ∧ (p ∨ r) and p ∧ (q ∨ r) ⇐⇒ (p ∧ q) ∨ (p ∧ r).
(distributive properties)

6. De Morgan’s Laws

(a) ¬(p ∨ q) ⇐⇒ (¬ p) ∧ (¬ q).


(b) ¬(p ∧ q) ⇐⇒ (¬ p) ∨ (¬ q)

7. p −→ q ⇐⇒ (¬ p) ∨ q.

8. ¬(p −→ q) ⇐⇒ p ∧ (¬ q).
9. p −→ q ⇐⇒ (¬ q) −→ (¬ p).

10. p ←→ q ⇐⇒ (p −→ q) ∧ (q −→ p):

2.3 Sets
One of the basic concepts every student of mathematics must know is that
of sets. Definition 9
A set is a well-defined collection of objects called elements.

A collection is well-defined if for any given object we can objectively decide whether it is or is
not in the collection. Any object which belongs to a given set is said to be an element of or a
member of the given set.

All Rights Reserved. 2020 Abdul, Atienza, et. al.


Lesson 2 29

Example 11.

1. The collection of all letters in the English Alphabet is a set.

2. The collection of all handsome guys is not a set, because one cannot objectively identify if
a given guy is handsome or not, because the word “handsome” is subjective in nature.

PUP DM S
Upper case letters are usually used to name sets. A set A can be commonly described in three
ways, by (a) listing (roster) method, (b) by set-builder notation or(c) by descriptive method. The
listing method describes the set by listing all the elements between braces and separated by
commas (note: in enumerating the elements of a certain set, each element is listed only once
and the arrangement of elements in the list is immaterial). The set-builder notation uses a
variable (a symbol, usually a letter, that can represent different elements of a set), braces, and
a vertical bar | that is read as "such that". This is usually used when the elements are too many
to list down. The descriptive method uses a short verbal statement to describe the set.

Example 12. Using the roster method, the set of months in a year that ends with letter ‘y’ can
be represented by {January, February, May, July}.

Example 13. The set {2; 3; 4; 5; 6; 7; 8; 9} in set-builder notation is

{x | x is an integer greater than 1 but less than 10}:

If a is an element of a set A, we write a ∈ A. Otherwise, we write a =∈ A. There are sets with


no elements. Such a set is said to be empty and we use the symbol ? to denote the empty set.
A set with only one element is called a unit set or a singleton.

Example 14. The set of integers between 1 and 2 is empty, while the set of even prime
numbers is a singleton.

For future discussion, we will use the following notations:

• N for the set of natural or counting numbers (positive integers): {1; 2; 3; 4;

:::} • Z for the set of integers: {::: − 4; −3; −2; −1; 0; 1; 2; 3; :::}

• Q for the set of • R for the set of


b| a; b ∈ Z; b 6=
rational numbers: real numbers 0
ff

a

All Rights Reserved. 2020 Abdul, Atienza, et. al.


Lesson 2 30

A set A is said to be finite if it is possible to list down all the elements of A in a list. Otherwise, A
is said to be infinite. If A is finite, the cardinality of A is the number of elements of A, which is
denoted by n(A).

Example 15. The set of all letters in the English Alphabet is finite and its cardinality is 26,
because there are 26 distinct letters in the English alphabet. On the other hand, the set of all
even integers in infinite.
PUP DM S
Definition 10
Let A and B be sets. We say that A is a subset of B and write A ⊆ B if every element of
A is an element of B. We say that A and B are equal and write A = B if A ⊆ B and B ⊆ A.

Remarks.

1. For any set A, A ⊆ A and ? ⊆ A.

2. If A and B are finite sets and A = B, then n(A) = n(B).

Example 16. Let A be the set of all mathematicians 20 feet high and B be the set of all PUP
students. Then A = ?. By Remark (1) above, A ⊆ B: Therefore, we can conclude that every
mathematician 20 feet high is a PUP student.

Two finite sets A and B are said to be equivalent if and only if n(A) = n(B). Note that equal sets
are necessarily equivalent bu equivalent sets need not be equal.

Example 17. Let A = {x | x is a prime number less than 20} and B = {1; 2; 3; 4; 5; 6; 7; 8} are
equiv alent since n(A) = 8 = n(B), however, A 6= B.

Definition 11
Let A and B be sets. The union of A and B is defined as

A ∪ B = {x | x ∈ A or x ∈ B}:

The intersection of A and B is

A ∩ B = {x | x ∈ A and x ∈ B}:

Then relative complement of B in A is the set

A \ B := {x ∈ A | x =∈ B}:
We could represent A ∪ B, A ∩ B, and A \ B in terms of Venn Diagrams as shown below. All

Rights Reserved. 2020 Abdul, Atienza, et. al.


Lesson 2 31

PUP
DMS
Example 18. Let A = {0; 1; 3; 5; 7} and B = {1; 2; 4; 7; 9}. Then A ∪ B = {0; 1; 2; 3; 4; 5; 7; 9}, A
∩ B = {1; 7} and A \ B = {0; 3; 5}.

In most of the interesting instances in mathematics, we normally talk about a particular set of
objects at a given time. The set of all objects of interest is called as the universal set,
generically denoted as U . If A ⊂ U , the complement of A is defined as the set

A0 = U \ A = {x ∈ U ∈ x =∈ A}:

Using the De Morgan’s Law of logic, one can readily verify that

(A ∪ B)0 = A0 ∩ B0and (A ∩ B)0 = A0 ∪ B0:

We can use our knowledge of sets to solve some word problems.

Example 19. At a certain high school, each student is a member of the English Club, the
Science Club, or the Mathematics Club. Of the 79 students asked, 33 are members of the
English Club, 37 are members of the Math Club, and 37 are members of the of the Science
club. Furthermore, 7 are members of both the English and the Math Clubs, 12 are members of
both the English and the Science Clubs, and 9 are members of the Science and Math Clubs.
No high school student is a member of all the three clubs. How many joined only the Math
Club?

Solution Let E, S, and M denote the sets of members of English, Science, and Mathematics
Club, respectively. As given in the problem, the universal set U has cardinality n(U ) = 79, n(E)
= 33, n(M) = 37, and n(S) = 37. Furthermore, n(E ∩ M) = 7, n(E ∩ S) = 12, and n(S ∩ M) = 9.
The last condition imply that E ∩ S ∩ M = ?. This situation can be represented by the following
Venn diagram.

All Rights Reserved. 2020 Abdul, Atienza, et. al.


Lesson 2 32

PUP
DMS
Since n(E ∩ S ∩ M) = 0, then the number of students who only joined the Mathematics

Club is n(M) − n(E ∩ M) − n(S ∩ M) = 37 − (7 + 9) = 21:

All Rights Reserved. 2020 Abdul, Atienza, et. al.

Lesson 2 33 Assessment

1. Write each statement in words. Let p: The plane is on time. Let q: The sky is clear.

(a) p ∧ (¬ q)
(b) q → (p ∨ ¬p)
(c) p ↔ q
PUP DM S
2. Construct a truth table for each proposition.

(a) [(p ∧ q) ∨ r] ↔ [(p ∧ r) ∨ (q ∧ r)]


(b) [(p ∧ r) → (q ∧ ¬r)] → [(p ∧ q) ∨ r)]

3. Prove the De Morgan’s Laws by constructing truth tables.

4. Let U := Letters in the English Alphabet = {a, b, c, . . . ,x, y, z}


A = {t, r, i, a, n, g, l, e, s}
B = {s, q, u, a, r, e };
C = {h, e, x, a, g, o, n, s }
Determine the following:

(a) A ∪ (B ∩ C)

(b) (A ∪ B)0 ∩ C
(c) (A ∩ C) ∪ (B ∩ C)

(d) A ∩ (C ∩ U )0
(e) n[(A ∪ B) ∩ (B ∪ C)]

5. A survey of 90 customers was taken at Barnes & Noble regarding the types of books
purchased. The survey found that 44 purchased mysteries, 33 purchased science fiction,
29 purchased romance novels, 13 purchased mysteries and science fiction, 5 purchased
science fiction and romance novels, 11 purchased mysteries and romance novels, and 2
purchased all three types of books (mysteries, science fiction, romance novels). How
many of the customers surveyed purchased

(a) mysteries only?


(b) mysteries and science fiction, but not romance novels?
(c) mysteries or science fiction?
(d) romance novels or mysteries, but not science fiction?
(e) exactly two types (mysteries, science fiction, romance novels)?

All Rights Reserved. 2020 Abdul, Atienza, et. al.

Lesson 3 34 Lesson 3: Problem Solving

Learning Outcomes

At the end of the lesson, the students are able to

1. differentiate between inductive and deductive reasoning;

PUP DM S
2. utilize inductive reasoning to form conjectures;

3. use deductive reasoning to prove a conjecture;

4. state the Polya’s four steps in problem solving;

5. solve mathematical problems using the Polya’s four steps.

3.1 Inductive and Deductive Reasoning


Human beings are said to be rational creatures because we use reasoning to come up with
sound decisions that we have to make everyday. Reasoning is our ability to use logical thinking
to come up with a decision. There are two major types of reasoning: inductive and deductive.
We first talk about inductive reasoning.

Definition 12
Inductive Reasoning is the process of reasoning that arrives at a general conclusion
based on the observation of specific examples.

Normally, we use inductive reasoning when we need to come up with a general conclusion,
known as a conjecture, by observing certain events or examples. Generally speaking, our
conjectures could be wrong. Examples which can negate our conjectures are called
counterexamples.

Example 20. In the past 30 days, we observed that the sun has risen in the east. Using
inductive reasoning, we may conjecture that the sun will rise in the east tomorrow.

Example 21. Consider the odd numbers 3; 5; 7; and 9. If we take their squares, we see that 32
= 9, 52 = 25, 72 = 49 and 92 = 81. We can observe that the squares of the given odd numbers
are all odd as well. Using inductive reasoning, we may conjecture that the square of an odd
integer is also odd.

Testing Conjectures
Logically speaking, we cannot prove a general statement from a number of specific examples
unless there are only finitely many examples and we can exhaust them. However, only one
counter example can prove

All Rights Reserved. 2020 Abdul, Atienza, et. al.


Lesson 3 35

that our conjecture is false.

Example 22. Let n be a positive integer. Select n distinct points at random in the
circumference of a circle and connect every pair of points in this collection by a chord.
Make a conjecture about the number of regions in the interior of the circle made by the
chords and test your conjecture.

For n = 1; 2; 3; 4; 5, we draw actual circles and count the number of regions made by

the chords obtained

PUP
DMS
by connecting every pair of points.

We summarize the number of regions in the following table.

n12345
no. of regions 1 2 4 8 16

If we observe the pattern on the number of regions, they seem to be powers of 2. In fact,
for n = 1; 2; 3; 4; 5, the number of regions in the circle is 2n−1. It is therefore reasonable
for use to give the following conjecture.

Conjecture. The number of regions in the interior of the circle made by connecting every
pair of points in a set of n points in the circumference is 2n−1.

The best way to test the conjecture is to check the example for the next larger n, which
is n = 6. Constructing the circle for n = 6 and counting the regions,
we see that the number of regions is 31 and not 26−1. This counterexample disproves

our conjecture. All Rights Reserved. 2020 Abdul, Atienza, et. al.

Lesson 3 36

Definition 13
Deductive reasoning is the process of reasoning that arrives at a conclusion based on
previously accepted general statements.

Deductive reasoning does not rely on examples. We make our conclusion based on general
statements whose truth value is known or assumed. Formal mathematics is usually based on
this type of reasoning. We first lay down definition of terms, and assume basic true statements

called axioms and derive true

PUP
DMS
statements from these axioms called as theorems.

Example 23. The following are examples of deductive reasoning.

1. Starfish are invertebrates. Patrick is a starfish. Therefore, Patrick is invertebrate. 2. Every


√ √
rational number is a real number. The number −1 is not real. Therefore, −1 is not rational.

Inductive reasoning cannot in general prove general statements as this relies on examples
only. In contrast, we can use deductive reasoning to prove a certain conjecture.

Example 24. Choose any number. Multiply by 3. Add 6 to the result. Divide the result by 3.
Finally, subtract the original number from the result of the previous step. Use inductive
reasoning to make a conjecture about the final result and use deductive reasoning to prove the
conjecture.

Solution. We first consider few examples.

test number 9 15 28
multiply by 3: 27 45 84
add 6: 33 51 90
divide by 3: 11 17 30
subtract the orig. no. 2 2 2

We see that based from the three test numbers, the final results are the same and are all equal
to 2. There is a reason to conjecture that the final result will always be 2 regardless on where
we start. To prove this claim, take an arbitrary number x.

multiply by 3: 3x
add 6: 3x + 6
divide by 3: 3x + 6

3= x + 2
subtract the orig. no: (x + 2) − x = 2.
Therefore, as claimed, it is now proven that we will always end up with 2.

All Rights Reserved. 2020 Abdul, Atienza, et. al.


Lesson 3 37

3.2 George Polya’s Guidelines for Problem Solving


In 1945, mathematician George Pólya devised a model for problem solving and published it in
his book How to Solve It. The book contains a collection of mathematical problems and
selected strategies on dealing these. His problem solving model, which he called heuristic (or
serving to discover), is as follows.

POLYA’S FOUR STEPS:

PUP DM S
1. Understand the problem. Ask questions, experiment, or otherwise rephrase the question
in your own words.

2. Devise a plan. Find the connection between the data and the unknown. Look for patterns,
relate to a previously solved problem or a known formula, or simplify the given
information to give you an easier problem.

3. Carry out the plan. Check the steps as you go.

4. Look back. Examine the solution obtained. In other words, check your

answer. Together with these guidelines, the following are some of his

recommended strategies:

1. Draw a diagram.
2. Solve a simpler 6. Find a pattern.

problem. 3. Make a 7. Use a formula or an

table. equation. 8. Using logical

4. Work backwards. reasoning.


5. Guess and check.
Example 25. In a seminar, 30 attendees were present. During their meet-and-greet activity,
they were asked to have a handshake with everyone in the room. If each one did handshake
with everyone, how may handshakes took place?

Solution.

Step 1. Understand the Problem. There were 30 attendees present. A simple handshake
means letting a distinct pair be recognized. Moreover, if A shakes hands with B, then B
shakes hands with A as well.

Step 2. Devise a plan. We start with solving simpler cases, say 3, 4 and 5 persons. We can
draw a diagram where a person is represented by nodes while handshakes by arcs
connecting the nodes. From here, we try to find a pattern.

Step 3. Carry out the plan. The following figures represent the handshakes that took place
among 3, 4 and 5 persons.

All Rights Reserved. 2020 Abdul, Atienza, et. al.


Lesson 3 38

PUP
DMS
A group of 3 persons makes 3 handshakes, a group of 4 persons makes 6 handsakes
and a group of 5 persons makes 10 handshakes. Now, for each case with k persons,
each of these persons has to have a handshake with the other k − 1 persons. So, the
product k(k − 1) is the number of all handshakes from individual perspective. Note that
if A shakes hands with B,
then B shakes hands with A as well. Thus, only half of k(k −1) represents the total
number of handshakes. Hence, a pattern is generalized by k(k − 1)

2will lead to the number of handshakes


that took place in a group of k persons. Therefore, there were a total of

k(k − 1)

2=30(30 − 1)

2= 435 handshakes.

Step 4. Look back. Every person will be shaking hands with 29 other. Thus, 870 handshakes
are noted for individual perperspective. Half of which is 435.

Example 26. Andrew has some magic cards to trade. Ian has 2 more than 2 times the number
of magic cards Andrew has. Patrick has 2 less than Ian. Ken has 4 less than 2 times the
number of magic cards Patrick has. Patrick has 8 magic cards. How many magic cards does
Andrew have to trade?

Solution.

Step 1. Understand the Problem. The number of magic cards Ian has depends on the number
of magic cards Andrew has. The number of magic cards Patrick has depends on the
number of magic cards Ian has. The number of magic cards Ken has depends on the
number of magic cards Patrick has. It is clear that 8 cards are in Patrick’s possession.

Step 2. Devise a plan. We can settle this by working backwards starting from the number of
magic cards Patrick has. Making a table may aid organization.

Step 3. Carry out the plan.

All Rights Reserved. 2020 Abdul, Atienza, et. al.


Lesson 3 39

Statements of Hints Arithmetic Sense Remarks


Patrick has 8 magic cards. 8 This is the last event.
Ken has 4 less than 2 times Operation is not
the number of magic cards (2 × 8) − 4 = 12 yet revered.
Patrick has. Ken has 12 cards.
Patrick has 2 less Operation is reversed.
less than Ian. 8 + 2 = 10 Ian has 10 cards.

PUP DM S
Ian has 2 more than Operation is reversed.
2 times the number of (10 − 2)=2 = 4 Andrew has 4 cards.
magic cards Andrew has.

Step 4. Look back. If Andrew has 4 magic cards, then Ian has 2 more than twice of 4 or 10
magic cards. If Ian has 10 magic cards, then Patrick has 10 - 2 = 8 magic cards.
Lastly, if Ken has 4 less than twice of 8 of 12 magic cards.
All Rights Reserved. 2020 Abdul, Atienza, et. al.

Lesson 3 40 Assessment

1. Explain why you can never be sure that a conclusion you arrived at using inductive
reasoning is true.

2. Select any two-digit number. Multiply it by 9. Then add the digits. Keep adding the digits in
the answer until you get a single-digit answer. Using inductive reasoning, what can you
conjecture

PUP

DMS
about any whole number multiplied by 9? Use deductive reasoning to prove that your
conjecture is true.

3. Use Polya’s Four Steps to solve the following problems.

(a) Susie’s age this year is a multiple of 5. Next year, her age is a multiple of 7. What is
her present age?

(b) Consider a square whose side is 1 unit. If the measure of its side is doubled, what
will be its new area as compare to the smaller square? How about if the side of the
smaller square was tripled, what will be its new area?

(c) How many perfect squares are there between 1,000,000 and 9,000,000?
(d) Determine the number of different triangles that can be drawn given eight noncollinear points?
(e) There are 25 students asked by their literature instructor regarding with the type of
literary works they prefer to read. He found out that 10 prefer to read novels, 11
prefer to read short stories, 15 prefer to read poems, 5 for both novels and short
stories, 4 both short stories and poems, 7 for both novels and poems, and 3 prefer
all. How many students prefer none of the given types of literary works?

All Rights Reserved. 2020 Abdul, Atienza, et. al.

Lesson 4 41 Lesson 4: Statistics and Data Management

Learning Outcomes

At the end of the lesson, the students are able to

1. demonstrate the ability to apply fundamental concepts in exploratory data analysis;


PUP

DMS
2. define the field of Statistics in terms of its definition and application;

3. enumerate the procedures involved in collecting data;

4. distinguish between the nominal, ordinal, interval and ratio methods of data

measurement; 5. recognize the various ways to present data;

6. identify the features that describe a data distribution.

Statistics is the study of the collection, organization, analysis, interpretation, and presentation
of data. It deals with all aspects of data, including the planning of its collection in terms of the
design of surveys and experiments. Some consider statistics a mathematical body of science
that pertains to the collection, analysis, interpretation or explanation, and presentation of data,
while others consider it a branch of mathematics concerned with collecting and interpreting
data. Because of its empirical roots and its focus on applications, statistics is usually
considered a distinct mathematical science rather than a branch of mathematics.

4.1 Basic Concepts

Statistics is defined as a branch of mathematics which is concerned with facilitating wise


decision making in the face of uncertainty and that, therefore develops and utilizes techniques
for collection, effective presentation, and proper analysis of data.

Branches of Statistics

1. Descriptive Statistics is concerned with the description and summarization of data, It


deals with the techniques used in the collection, presentation, organization, and analysis
of the data on hand.

2. Inferential Statistics is concerned with the drawing of conclusions from data. It deals with
the techniques used in generalizing from samples to populations, performing estimations
and hypothesis tests determining relationships among variables, and making predictions.

All Rights Reserved. 2020 Abdul, Atienza, et. al.


Lesson 4 42

Functions of Statistics

1. Condensation. Generally speaking by the verb ‘to condense’, we mean to reduce or to


lessen. Condensation is mainly applied at embracing the understanding of a huge mass
of data by providing only few observations.

2. Comparison. Classification and tabulation are the two methods that are used to condense
the data. They help us to compare data collected from different sources. Grand totals,
measures

PUP

DMS
of central tendency measures of dispersion, graphs and diagrams, coefficient of
correlation, etc. provide ample scope for comparison. As statistics is an aggregate of
facts and figures, comparison is always possible and in fact comparison helps us to
understand the data in a better way.

3. Forecasting. By the word forecasting, we mean to predict or to estimate beforehand.


Given the data of the last ten years connected to the number of students enrolled in PUP,
it is possible to predict or forecast the number of students that will enroll for the near
future. In business also forecasting plays a dominant role in connection with production,
sales, profits etc. The analysis of time series and regression analysis plays an important
role in forecasting.

4. Estimation. One of the main objectives of statistics is drawn inference about a population
from the analysis for the sample drawn from that population.

5. Tests of Hypothesis. A statistical hypothesis is some statement about the probability distri
bution, characterizing a population on the basis of the information available from the
sample observations. In the formulation and testing of hypothesis, statistical methods are
extremely use ful. Whether the grades of students increased because they are motivated
or whether the new teaching method is effective in discussing a particular topic are some
examples of statements of hypothesis and these are tested by proper statistical tools.

Scope of Statistics

1. Statistics and Industry. Statistics is widely used in many industries. In industries, control
charts are widely used to maintain a certain quality level. In production engineering, to
find whether the product is conforming to specifications or not, statistical tools, namely
inspection plans, control charts, etc., are of extreme importance. In inspection plans we
have to resort to some kind of sampling - a very important aspect of Statistics.

2. Statistics and Commerce. Statistics are lifeblood of successful commerce. Any


businessman cannot afford to either by under stocking or having overstock of his goods.
In the beginning he estimates the demand for his goods and then takes steps to adjust
with his output or purchases. Thus statistics is indispensable in business and commerce.

All Rights Reserved. 2020 Abdul, Atienza, et. al.


Lesson 4 43

3. Statistics and Economics. Statistical methods are useful in measuring numerical changes
in complex groups and interpreting collective phenomenon. Nowadays the uses of
statistics are abun dantly made in any economic study. Both in economic theory and
practice, statistical methods play an important role.

4. Statistics and Education. Statistics is widely used in education. Research has become a
common feature in all branches of activities. Statistics is necessary for the formulation of
policies

PUP

DMS
to start new course, consideration of facilities available for new courses etc. There are
many people engaged in research work to test the past knowledge and evolve new
knowledge. These are possible only through statistics.

5. Statistics and Planning. Statistics is indispensable in planning. In the modern world, which
can be termed as the “world of planning”, almost all the organizations in the government
are seeking the help of planning for efficient working, for the formulation of policy
decisions and execution of the same. In order to achieve the above goals, the statistical
data relating to production, consump tion, demand, supply, prices, investments, income
expenditure etc and various advanced statistical techniques for processing, analyzing and
interpreting such complex data are of importance. In India statistics play an important role
in planning, commissioning both at the central and state government levels.

6. Statistics and Medicine. In Medical sciences, statistical tools are widely used. In order to
test the efficiency of a new drug or medicine, t - test is used or to compare the efficiency
of two drugs or two medicines, t-test for the two samples is used. More and more
applications of statistics are at present used in clinical investigation.

7. Statistics and Modern Applications. Recent developments in the fields of computer


technol ogy and information technology have enabled statistics to integrate their models
and thus make statistics a part of decision making procedures of many organizations.
There are so many software packages available for solving design of experiments,
forecasting simulation problems etc.

Limitations of Statistics

1. Statistics is not suitable to the study of qualitative phenomenon. Since statistics is


basically a science and deals with a set of numerical data, it is applicable to the study of
only these subjects of enquiry, which can be expressed in terms of quantitative
measurements. As a matter of fact, qualitative phenomenon like honesty, poverty, beauty,
intelligence etc, cannot be expressed numerically and any statistical analysis cannot be
directly applied on these qualitative phenomenon.

2. Statistics does not study individuals. Statistics does not give any specific importance to
the individual items; in fact it deals with an aggregate of objects. Individual items, when
they are taken

All Rights Reserved. 2020 Abdul, Atienza, et. al.


Lesson 4 44

individually do not constitute any statistical data and do not serve any purpose for any
statistical enquiry.

3. Statistical laws are not exact. It is well known that mathematical and physical sciences are
exact. But statistical laws are not exact and statistical laws are only approximations.
Statistical conclusions are not universally true. They are true only on an average.

PUP DM S
4. Statistics table may be misused. Statistics must be used only by experts; otherwise,
statistical methods are the most dangerous tools on the hands of the inexpert. The use of
statistical tools by the inexperienced and untraced persons might lead to wrong
conclusions.

5. Statistics is only one of the methods of studying a problem. Statistical method do not
provide complete solution of the problems because problems are to be studied taking the
background of the countries culture, philosophy or religion into consideration. Thus the
statistical study should be supplemented by other evidences.

Population and Sample

In statistics, we are often interested in gathering information from a group of objects. If the
group in consideration consists of large number of objects, we try to obtain information about
the group by examining its subgroup.

Definition 14
The total collection of all the elements that we are interested in is called a population. A
subgroup of the population that will be studied in detail is called a sample.

In order for the data from the sample is informative about the population, it must be
representative of the population. Being representative of the population does not mean that the
characteristic of the sample is exactly that of the total population, but instead the sample was
obtain in such way that every member of the population had an equal chance to be included in
the sample.

Definition 15
A sample of k members of a population is called a random sample, also called a simple
random sample, if the members are chosen in such a way that all possible choices of the
k members are equally likely.

After a random sample is obtain from the population, we can use statistical inference to draw
general izations about the population by examining the members of the sample.

All Rights Reserved. 2020 Abdul, Atienza, et. al.


Lesson 4 45

4.2 Steps in Statistical Investigation


1. Defining the problem

(a) Identify a specific problem.


(b) Define the scope and limitations, assumptions to be made, and expected outcomes.

2. Collection of data

PUP DM S
(a) Make sure to collect the data properly.
(b) Incomplete, fabricated, outdated, and inaccurate data are useless.

3. Summarization and tabulation of data

(a) This refers to organization of data in text, tables, graphs and charts, so that logical
conclusion can be derived from them.
(b) Explore the data to obtain additional insight that could contribute to the

study. 4. Analysis of data

(a) This pertains to the process of deriving from the given data relevant information from
which numerical descriptions can be formulated.
(b) Summarized data must be examined so that insights and meaningful information ca
be pro duced to support decision-making or solutions to the question or problem at
hand.

5. Interpretation of data and results

(a) Refers to the task of drawing conclusions from the analyzed data.
(b) Results must be able to answer the research problem and give

recommendations. 6. Presentation of the result

(a) Present all pertinent results in a clear and concise manner.


(b) Use appropriate form of media to present results.

4.3 Sampling and Sampling Techniques


Sampling refers to the process of obtaining samples from the population. Sampling maybe
categorized as either probability sampling or non-probability sampling. Probability sampling,
also referred to as random sampling, is the method of sampling in which every member of the
population have equal chance of being selected as sample; otherwise, it is considered as
non-probability sampling. We should note that in able to properly use the techniques of
statistical inference, probability sampling must be used to obtain samples.

All Rights Reserved. 2020 Abdul, Atienza, et. al.


Lesson 4 46

Probability Sampling Techniques

1. Simple Random Sampling. A probability sampling technique wherein all possible subsets
con sisting of n elements selected from the N elements of the population have the same
chances of selection.

2. Systematic Sampling. This is a probability sampling technique wherein the selection of the
first element is at random and the selection of other elements in the sample is systematic

by

PUP
DMS
subsequently taking every kth element from the random start where k is the sampling interval.

3. Stratified Random Sampling. A probability sampling method where we partition the


population into non-overlapping strata or group and then a proportional sample is chosen
from each strata. The actual sample is the sum of the samples derived from each strata.

4. Cluster Sampling. A probability sampling technique wherein we partition the population


into non-overlapping groups or clusters consisting of one or more elements, and then
select a sample of clusters. Every member of the selected cluster will be considered as
sample.

Non-Probability Sampling Techniques

1. Accidental Sampling. Sample is chosen by the researcher by the obtaining members of


the population in a convenient, often haphazard way.

2. Quota Sampling. There is specified number of persons of certain types is included in the
sample. The researcher is aware of categories within the population and draws samples
from each category. The size of each categorical sample is proportional to the proportion
of the population that belongs in that category.

3. Purposive Sampling. The researcher employs his or her judgments on choosing which he
or she believes are representative of the population.

4. Snowball Sampling. This technique is also called referral sampling. A primary set of
samples are chosen based on the criteria set by the researcher. Information on where to
find succeeding set of sample having the same criteria will be gathered from this primary
set in order to expand the number of samples.
4.4 Sample Size Considerations
The sample size is typically denoted by n and it is always a positive integer. No exact sample
size can be mentioned here and it can vary in different research settings. However, all else
being equal, large sized sample leads to increased precision in estimates of various properties
of the population. To determine the sample size we can apply one of the following methods:

All Rights Reserved. 2020 Abdul, Atienza, et. al.


Lesson 4 47

1. Slovin’s Formula. Slovin’s formula is used to calculate the sample size n given the
population size and a margin of error E. It is a formula use to estimate sampling size of a
random sample from a given population. We can compute

n =N
2
1 + NE ;

where N is the population size.

PUP DM S
Example 27. A researcher plans to conduct a survey about food preference of BS Stat
students. If the population of students is 1000, use the Slovin’s formula to find the sample size
if the margin of error is 5%.

Solution. Using the Slovin’s formula, we get

n =1000
2
1 + 1000(0:05) ≈ 285:71:

Therefore, the researcher needs to survey 286 BS Stat Students.

2. Minimum Sample Size for Estimating a Population Mean. The estimated minimum sample
size n needed to estimate a population mean — to within E units at 100(1 − ¸)% confidence
is

2 2
n =(z¸=2) ff
2
E ;
where ff is the known population standard deviation, E is the margin of error and z¸=2 is a
value which can be obtained in the z-table.

Example 28. Suppose we want to know the average age of STEM students. We would like to
be 99% confident about our results. From previous study, we know that the standard deviation
for the population is 1.3. How many students should be chosen for a survey if the margin of
error is 0.2.

Solution. Find z¸=2 by looking at the z-table.

¸ = (1 − 0:99) = 0:01 =⇒ z¸=2 = z0:005:

The closest z-score for 0:005 in the z-table is 2:58. Thus,

2 2
n =(2:58) (1:3)
2
(0:2) ≈ 281:23:

which we round up to 282, since it is impossible to take a fractional observation. We need a


282 STEM students as a sample for our study.

All Rights Reserved. 2020 Abdul, Atienza, et. al.


Lesson 4 48

3. Minimum Sample Size for Estimating a Population Proportion The estimated minimum
sample size n needed to estimate a population proportion p to within E at 100(1−¸)%
confidence
is

2 2
n =(z¸=2) pˆ(1 − pˆ) E :
This is also called the Cochran Formula.
PUP DM S
The dilemma here is that the formula for estimating how large a sample to take contains
the number pˆ, which we know only after we have taken the sample. There are two ways
out of this dilemma.

• First, typically the researcher will have some idea as to the value of the population
proportion p, hence of what the sample proportion pˆ is likely to be. For example, if
last month 37% of all voters thought that state taxes are too high, then it is likely that
the proportion with that opinion this month will not be dramatically different, and we
would use the value 0.37 for pˆ in the formula.

• The second approach to resolving the dilemma is simply to replace pˆ in the formula
by 0.5. This is because if pˆ is large then 1 − pˆ is small, and vice versa, which limits
their product to a maximum value of 0.25, which occurs when pˆ = 0:5. This is called
the most conservative estimate, since it gives the largest possible estimate of n.

Example 29. Suppose we are doing a study on the inhabitants of a large town, and want to find
out how many households serve breakfast in the mornings. We don’t have much information on
the subject to begin with, so we’re going to assume that half of the families serve breakfast:
this gives us maximum variability. Here, pˆ = 0:5. We want 95% confidence and at least 5%
precision.

Solution. Find z¸=2 in the z-table. We have

¸ = (1 − 0:95) =⇒ z¸=2 = z0:025:

The closest z-score for 0:025 in the z-table is 1:96. A 95% confidence level gives us Z values of
1.96,
we get
2
n =(1:96) (0:5)(1 − 0:5)
2
(0:05) ≈ 384:16:
Hence, a random sample of 385 households in our target population should enough to give us
the confidence levels we need.

All Rights Reserved. 2020 Abdul, Atienza, et. al.


Lesson 4 49

Finite Population Correction for Proportions

If the population is small then the sample size can be reduced slightly. This is because a given
sample size provides proportionately more information for a small population than a large
population. The formula
is
1 +n0 − 1
N
n =n0
;

PUP DM S
where n0 is the Cochran’s sample size recommendation, N is the population size and n is the
new adjusted sample size.

Example 30. In the preceding example, if there were just 1000 households in the target
population, we
would calculate 1000
≈ 278:18:
n =385 1 +385 − 1

All we need are 279 households in our sample, a substantially smaller sample size.

4.5 Methods of Data Collection


1. Survey Method. The survey is a method of collecting data on the variable of interest by
asking people questions. This may be done, by interview or by using questionnaires.

2. Observation. Observation is a method of obtaining data or information by using our


primary senses.

3. Experiment. Experiment is a method of collecting data where there is direct human


intervention on the conditions that may affect the values of the variable of interest.

4.6 Levels of Measurement


1. The nominal level of measurement classifies data into mutually exclusive
(non-overlapping) categories in which no order or ranking can be imposed on the data.

Example: Gender (male, female), Zip Code, Color, Nationality, Political affiliation,
Religious affiliation.

2. The ordinal level of measurement classifies data into categories that can be ranked;
however, precise differences between the ranks do not exist.

Example: Grade(A,B,C,D,F), Rating Scale/Likert scale, Ranking of tennis players,


Judging (First place, second place, etc.

All Rights Reserved. 2020 Abdul, Atienza, et. al.


Lesson 4 50

3. The interval level of measurement ranks data, and precise differences between units of
measure do exist; however, there is no meaningful zero.

Example: Temperature, IQ, SAT score

4. The ratio level of measurement possesses all the characteristics of interval measurement, and
there exists a true zero. In addition, true ratios exist when the same variable is measured
on two

PUP
DMS
different members of the population

Example: Height, Weight, volume, Time, Salary, Age

4.7 Presentation of Data


After data have been collected, the researcher can now present them in the following logical
methods.

1. Textual Form. Data are presented in paragraph of text. The text highlights the important
figures or results that the researcher wishes to focus on.

2. Tabular Form. Data appears in a systematic manner in rows and


columns. The following is an example of a Simple or One-Way Table.

Table 1
Frequency Distribution of the
Students Enrolled for the Last 6 Years

Year Frequency
2012 13,450
2013 13,200
2014 15,389
2015 16,790
2016 18,900
2017 19,500
Total 97,229

All Rights Reserved. 2020 Abdul, Atienza, et. al.


Lesson 4 51 The following is an example of a Two-Way Table.

Table 2
Number of Students Enrolled for the Last 6 Years
When Grouped According to Sex

SexYear

PUP DM S
2012 2013 2014 2015 2016 2017 Total
Male 5560 6095 7386 8056 7945 6451 41493
Female 7890 7105 8003 8734 10955 13049 55736
Total 13450 13200 15389 16790 18900 19500 97229

3. Graphical Form. Data or relationship among variables could be presented in visual form,
thru graph or diagrams. In that manner, the reader can easily perceive what is being
meant by the figure or any trend being portrayed by the data.

Types of Statistical Charts

(a) Bar Graph (Vertical Bar/Column Charts) is applicable for showing comparison of
amount of a variable of interest collected over time.

Simple Chart

Grouped Column Charts

All Rights Reserved. 2020 Abdul, Atienza, et. al.


Lesson 4 52 Subdivided Column Charts
PUP DM S
(b) Histogram is similar to the bar graph but the base of the rectangle has a length
exactly equal to the class width of the corresponding interval. Also, there are no
spaces between rectangles.

Histogram

(c) Pictograph is similar to the bar chart but instead of bars, we use pictures or symbols
to represent a value or an amount.

Pictograph
(d) Pie Chart is a circular graph partitioned into several section, depicting relative
percentage with respect to the total distribution.

All Rights Reserved. 2020 Abdul, Atienza, et. al.


Lesson 4 53 Pie Chart

PUP DM S
(e) Line Graph is a graph used to visualize data that changes continuously over

time. Simple Line Graph

Multiple Line Graph


(f) Statistical Map is used to show data in geographical areas.

Statistical Map

All Rights Reserved. 2020 Abdul, Atienza, et. al.


Lesson 4 54

4.8 Measures of Central Tendency


A measure of central tendency or average is a location measure that pinpoints the center or
typical middle value of a data set. A convenient way of describing a set of data with a value
that describes the average characteristic a data set. The three common measures of central
tendency are the mean, median and mode.

Mean

DMS
PUP
Definition 16
Suppose that a variable x assumes values x1; x2; : : : ; xn. The arithmetic mean x of these
values
is defined as 1 Xn
n= n i=1 n:
x=
P
xi =x1 + x2 + · · · + xn
x

The (arithmetic) mean of x is obtained by adding all its observed values and dividing the sum
by the total number of observations.

Example 31. The scores of 15 students in Mathematics in the Modern World on an exam
consisting of 25 items are 25,20,18,18,17,15,15,15,14,14,13,12,12,10,10. Determine the mean
score for this exam.

Solution. Let x denote the score of a random student from the sample of 15 students in
P
Mathematics in the Modern World. The sum of these scores is x = 228. Hence, the mean
score of the 15 students is
P
x

n=228
x=

15= 15:2:

There are cases when the observations in a data set assume respective weights. In this case
where the weights are positive integers, we can call these weights as frequencies. The
following gives a formula for the weighted mean of a weighted data set.

Definition 17
Given the x values x1; x2; : : : ; xn assuming respective weights w1; w2; : : : ; wn, the weighted
mean
is defined as P
x=w1x1 + w2x2 + · · · + wnxn w1
x=
P + w2 + · · · + wn:
wx

Example 32. Suppose that we are asked to get the mean of the data set 1; 1; 3; 3; 3; 3; 4; 4; 4;

6; 6; 8. All Rights Reserved. 2020 Abdul, Atienza, et. al.

Lesson 4 55
Using the original formula for the arithmetic mean we find that

x =(1 + 1) + (3 + 3 + 3 + 3) + (4 + 4 + 4) + (6 + 6) + 8
12

=2 · 1 + 4 · 3 + 3 · 4 + 2 · 6 + 1 · 8
1+4+3+2+1

=2 + 12 + 12 + 12 + 8
46 12
= 12

PUP DM S
= 3:833

We can interpret the mean of the data values as the fulcrum or center of gravity in a balance
scale as shown below.

12345678

mean = 3:8333

Example 33.
Calculate the General Weighted Average corresponding weights wi. By
(GWA) of Julius Garde for the first constructing a fourth column consisting of
semester of school year 2019-2020 as the products wixi and finding the column
shown in the following table. totals, we get the table below.

Course Grade Units BM 112 1.25 3 BM


Solution. To solve for the GWA, we first 101 1.00 3 AC 103 1.25 6 MG 101 1.00 3
consider the entries on the second
EC 111 1.50 3 MK 101 1.50 3 FM 111
column of the table as the points xi and
1.20 3 PE 1 1.00 2
the entries in the third column as the

All Rights Reserved. 2020 Abdul, Atienza, et. al.


Lesson 4 56

Course xi wi wixi
BM 112 1.25 3 3.75
BM 101 1.00 3 3.00
AC 103 1.25 6 7.50
MG 101 1.00 3 3.00
EC 111 1.50 3 4.50
MK 101 1.50 3 4.50

PUP DM S
FM 111 1.20 3 3.60
PE 1 1.00 2 2.00
P P
Total w = 26 wx = 32:00
P P
We see from the column totals that w = 26 and wx = 32. Therefore, the weighted mean or
the general weighted average (GWA) of Julius Garde for the first semester of AY 2019-2020 is
P
wx
P 32
w= 26= 1:23:
Definition 18
x=
Median

The median, usually denoted by x˜, is the middle value of a data set if the observations
are arranged either in increasing or decreasing order.

Outliers in the data set do not affect the median. Thus, the median is preferred over the mean
as a measure of central tendency when the data contains outliers. To find the median, begin by
listing the data in order from smallest to largest, or largest to smallest.

If the number of data values, N, is odd, then the median is the middle data value. This value
can be found by rounding N=2 up to the next whole number. If the number of data values is
even, there is no one middle value, so we find the mean of the two middle values (values N=2
and N=2 + 1)

Example 34. Given the scores of 15 students in Mathematics in the Modern World on an exam
consisting of 25 items:
25; 20; 18; 18; 17; 15; 15; 15; 14; 14; 13; 12; 12; 10; 10

Since the data is already arranged in decreasing order and there are 15 observations, hence,
15
we round up 2= 7:5 to the nearest whole number, which is 8, and take the 8th observation
from the left (or right). Therefore, the median is x˜ = 15: In comparison to example 31, the
computed mean is 15:2.

All Rights Reserved. 2020 Abdul, Atienza, et. al.


Lesson 4 57

2
PUP DM S
1

10 11 12 13 14 15 16 17 18 19 20 21 22 23 24 25 26 mean

median

Remark. In general, the median need not equal the mean.

Example 35. The data given below is the total number of hours lost due to tardiness and
absences of employees in a company in a given year. Find the median.
we take note of the two middle
Month Hours Lost
January 55 February 23 March 24
observations then compute
April 37
May 37
June 48
July 42
August 27 September 20 October 40 November 30 December
32
Solution. If the data are arranged in

increasing order, we have 20; 23; 24; 27;


x˜ =32 + 37
30; 32; 37; 37; 40; 48; 42; 55: 2= 34:5:

Since there are 12 observations (even),

Therefore, the median number of hours lost due to tardiness and absences of employees in a
company in the given year is 34:5 hours.

Mode

Definition 19
The mode is the most frequent observation in a given data set.

Outliers in the data set do not affect the mode. It is possible that the mode of a data set does
not exist, and it is not always unique. It is an appropriate measure of average for data
measured only in the nominal level. We will denote mode using the symbol xˆ.

All Rights Reserved. 2020 Abdul, Atienza, et. al.


Lesson 4 58

Example 36. Suppose that we wanted to know the “average color” of cars used by the
residents in a given village. In our vehicle color survey, we collected the following data.

Color Frequency
Blue 3
Green 5
Red 4
White 3

PUP DM S
Black 2
Grey 3
Since color of vehicles are measured up to the nominal level, the most appropriate measure for
the “average color” is then the mode. The most frequent color is Green, a total of 5 vehicles.
Therefore, the “average color” in our survey data must be Green.

It is possible for a given data set to have more than one modes. Such a data set is said to be
multimodal. If a given set has only one mode, the data set is unimodal. If it has two modes, the
data set is bimodal, and so on.

4.9 Measures of Dispersion or Variability


Measures of dispersion are descriptive summary measures that helps us characterize the data
set in terms of how varied the observations are from the center. If its value is small, then this
indicates that the observations are not too different from the center. On the other hand, if its
value is large, then this indicates that the observations are very different from the center or that
they are widely spread out from the center.

Range

Definition 20
The range is the difference between the largest and the smallest observations or items in
a set of data.

The range of a data set is easy to compute, but it is a limited measure because it depends on
only two of the numbers (the highest and the lowest) in the data set. Hence, the range can
easily be affected by outliers. Also, it does not provide any information regarding the
concentration of the data from the center.

Example 37. The following are scores of 20 coming from two different sections, 10 from each
section, in a 50-item exam in MMW.
section 1 40 38 42 40 39 39 43 40 39 40
section 2 46 37 40 33 42 36 40 47 34 45

All Rights Reserved. 2020 Abdul, Atienza, et. al.


Lesson 4 59 For section 1, the highest score is 43, while the lowest score is 38. Thus,

range = 43 − 38 = 5:

On the other hand, for section 2, the highest score is 47, while the lowest score is 33. Thus,

range = 47 − 33 = 14:

PUP DM S
Therefore, the scores of students surveyed from section 2 gets a wider range than those of
students surveyed from section 1.

Variance and Standard Deviation


Suppose that the center of a population data set {x1; x2; : : : ; xN} is best described by the
arithmetic mean — and that our goal is to get the average “distance” of each data point xiform
—. Naturally, we would like to compute for
N
1 X i=1 (xi −
N —):

However, using the properties of summations, and the fact that n— = x1 + x2 + · · · + xN we can
check
that X N— = 0:
N (xi − —) = N XN
X i=1 i=1 xi − i=1
— = N— −

In other words, the sum of the deviations from the mean is 0, and therefore, we cannot have a
meaningful measure of variability this way. The reason behind this fact is that some of the
deviations from the mean are negative (those which are to the left of the mean) and some are
positive (those which are to the right of the mean) and they cancel each other out. However,
we can work our way out of this unfortunate situation if we can ignore the signs of these
deviations. One way to do this is to take the square these deviations from the mean. We then
have the following definition.

Definition 21
The variance of a population data set {x1; x2; : : : ; xN} with population mean — is defined as
1

1 X 2
ff2 = N N (xi − —) :
i=

On the other hand, the variance of a sample data set {x1; x2; : : : ; xn} with sample mean x is
defined as n (xi − —)2:
X i=1
s2 =1 n − 1

As we may have noticed, the formula for the sample variance differs significantly from the

formula for All Rights Reserved. 2020 Abdul, Atienza, et. al.

Lesson 4 60

the population variance mainly because of the divisor n − 1. The reason behind this is rather
technical and mathematical in nature. Simply taken, the divisor n − 1 removes the “bias” in s2
when we want it to estimate ff2for the purposes of making inferences.

Notice that the variance is a nonnegative quantity because it came from averaging squared
quantities.
We also realize that there is one major drawback to using the variance. If we follow the steps in

calcu

PUP
DMS
lating the variance, we find that the variance is measured in terms of square units because we
took the squares of the deviation. For example, if our sample data is measured in terms of
meters, then the units for a variance would be given in square units.

In order to standardize the units, we can take the square root of the variance to eliminate the
problem of squared units, and gives us a measure of the spread that will have the same units
as our original sample or population data.

Definition 22
The population (sample) standard deviation is the nonnegative square root of the the pop
ulation (sample) variance. In symbols,
√ 2 √ 2
ff = ff and s = s :
Example 38. Using the sample data sets in example 37, determine which section exhibits a
greater variability in terms of standard deviations.

Solution. Let x denote the scores of students sampled from section 1 and let y denote the
scores of students sampled from section 2. To calculate the standard deviations of each
sample, we first take note that the sample means from each section are

n=400 n=400
x=
P
x 10= 40 and y = 10= 40:
P
y

To calculate the sample standard deviation, we construct the following table.

All Rights Reserved. 2020 Abdul, Atienza, et. al.


Lesson 4 61

x y x − x y − y (x − x)2(y − y)2
40 46 0 6 0 36
38 37 −2 −3 4 9
42 40 2 0 4 0
40 33 0 −7 0 49
39 42 −1 2 1 4
39 36 −1 −4 1 16

PUP DM S
43 40 3 0 9 0
40 47 0 7 0 49
39 34 −1 −6 1 36
40 45 0 5 0 25
P P P P
x = 400 y = 400 (x − x)2 = 20 (y − y)2 = 224 Therefore, the sample

variance for the sample from section 1 is


P
(x − x)2
s2 = 2:2222;
20
n − 1= 9=
while the sample variance for the sample from
P
section 2 is (y

− y)2

s2 = 24:8888:
224
n − 1= 9=
Taking square roots, we find that the sample standard deviations of section 1 and section 2
√ √
respectively are 2:2222 ≈ 1:49 and 24:8888 ≈ 4:99. We can conclude that for these samples,
the one from section 1 exhibits the lesser variability than that from section 2. We comment that
even though the two samples have equal means, the standard deviations showed the actual
difference between the two data sets.

All Rights Reserved. 2020 Abdul, Atienza, et. al.

Lesson 4 62 Assessment

1. A research objective is presented. For each,identify the (a)population and (b) sample in the
study.
(a) A polling organization contacts 2141 male university graduates who have a
white-collar job and asks whether or not they had received a raise at work during the
past 4 months.
(b) A quality-control manager randomly selects 70 bottles of ketchup that were filled on

July 17

PUP
DMS
to assess the calibration of the filling machine.
(c) Every year the PSA releases the Current Population Report based on a survey of
50,000 households. The goal of this report is to learn the demographic
characteristics, such as income, of all households within the Philippines.

2. Determine the level of measurement of each variable.

(a) birth order among siblings in a family


(b) favorite movie
(c) volume consumption of water used by a household in a day
(d) eye color
(e) number of siblings

3. Determine the type of sampling used.


(a) A member of Congress wishes to determine her constituents’ opinion regarding
estate taxes. She divides her constituency into three income classes: low-income
households, middle-income households, and upper-income households. She then
takes a simple random sample of house holds from each income class.
(b) A college official divides the student population into five classes: freshman,
sophomore, junior, senior, and graduate student. The official takes a simple random
sample from each class and asks the members opinions regarding student services.
(c) The presider of a guest-lecture series at a university stands outside the auditorium
before a lecture begins and hands every fifth person who arrives, beginning with the
third, a speaker evaluation survey to be completed and returned at the end of the
program.
(d) To determine his DSL Internet connection speed, Shawn divides up the day into four
parts: morning, midday, evening, and late night. He then measures his Internet
connection speed at 5 randomly selected times during each part of the day.
(e) 24 Hour Fitness wants to administer a satisfaction survey to its current members.
Using its membership roster, the club randomly selects 40 club members and asks
them about their level of satisfaction with the club.

All Rights Reserved. 2020 Abdul, Atienza, et. al.


Lesson 4 63

4. Patricia categorized her spending for this month into four categories: Rent, Food, Fun,
and Other. The percents she spent in each category are pictured here. If she spent a total
of PhP 26,000 this month, how much did she spend on rent?

PUP DM S
5. You recorded the time in seconds it took for 8 participants to solve a puzzle. The times
were: 15.2, 18.8, 19.3, 19.7, 20.2, 21.8, 22.1, 29.4.

(a) Calculate the mean and the median time it took for the 8 participants to solve a puzzle.
(b) Calculate the range and standard deviation of the time it took for the 8 participants to
solve the puzzle.

6. Make up three data sets with 5 numbers each that have:

(a) the same mean but different standard deviations.


(b) the same mean but different medians.
(c) the same median but different means.

All Rights Reserved. 2020 Abdul, Atienza, et. al.

Lesson 5 64 Lesson 5: Financial Mathematics

Learning Outcomes

At the end of the lesson, the students are able to

1. to compute the accumulated or present value of a certain amount of money


PUP

DMS
2. to compute the accumulated or present value of a series of periodic payments

(annuity) 3. to construct an amortization schedule when paying off a loan

Managing your money takes much thought and planning. All your daily needs must be met, but
at the same time you should consider your long-term goals such as purchasing a house,
saving for college expenses, and investing for retirement.
A component that is common to all financial transactions is the investment of money at interest.
When a bank lends money to you, it charges rent for the money. When you lend money to a
bank (also known as making a deposit in a savings account), the bank pays rent to you for the
money. In either case, the rent is called ”interest”.

Understanding the cost of borrowing money will help you make informed decisions about your
personal finances.

5.1 Simple and Compound Interest


The money a bank or other lender is willing to lend you is called the amount of credit extended
or the principal of the loan. The amount of credit and the interest rate that you may obtain
depend on the assurance you can give the lender that you will be able to repay the loan based
on any of the following (1) your credit reputation, (2) your capacity to earn money or income or
(3) the security or collateral you pledge to be paid to the lender in case you default in your
payment.

Banks sometimes grant loans even without security, but they require the signature of one or
more other persons, called cosigners or co-debtors, who guarantee the loan will be repaid. For
either of the two types of loans, the secured loan or the cosigner loan, the borrower (and
cosigner, if there is one) must sign an agreement called a personal note or a bank note. This
document states the terms and conditions of the loan.

The most common ways of borrowing money is through

All Rights Reserved. 2020 Abdul, Atienza, et. al.


Lesson 5 65

• a loan paid in periodic installments,

• a credit card,

• a depository account in banks (banks are the borrowers), and

• a stock or a bond (gov’t and private corporations sell bonds to raise funds).
PUP

DMS
The concept of simple interest is essential to the understanding of short-term transactions
(payment/s is made within a year). Interest is the money the borrower pays for the use of the
lender’s money. One type of interest is called simple interest.

Definition 23: Simple Interest

Simple interest is based on the entire amount of the loan for the total period of the loan.
The formula used to find simple interest follows.

Interest = principal × rate × time


I=Prt

where P is the principal amount, r is the interest rate and the nonnegative value of t (t ≥
0) is the term of the investment (number of days, months or years for which the money
will be lent).

Since simple interest is usually assumed for short-term transactions, the time

t =# of days between two dates


# of days in a year
is considered to be a fraction of a year and is determined under the following different
conventions:

• Exact Interest. Exact number of days between dates is used and the number of days in a
year is taken to be 365 days (366 days for leap years).

• Ordinary Interest. Each month is said to have 30 days and the number of days in a year is
taken to be 360 days (i. e. 12 × 30).

• Banker’s Rule or Merchant’s Rule. Exact number of days between dates is used and the
num ber of days in a year is taken to be 360 days. Under this rule, an investor or lender
mathematically yields the highest amount of interest so banks would prefer this method.

Example 39. Suppose that Feli borrowed PhP 25,000 from the Provident Fund on March 8 and
pays the entire sum including interest on October 3 of the same year, and that the interest rate
is 5%. Find the amount of interest earned, if it is computed using (a) exact simple interest (b)
ordinary simple interest

All Rights Reserved. 2020 Abdul, Atienza, et. al.


Lesson 5 66 and (c) banker’s rule. Assume non leap year.

Solution: From March 8 to October 3, there are exactly N = 23 + 30 + 31 + 30 + 31 + 31 + 30 +


3 = 209 days and under ordinary interest, there are N = 22 + 30 + 30 + 30 + 30 + 30 + 30 + 3
days. Thus the amount of interest is

(a) I = 25; 000 × 0:05 ×209


365 = PhP 715.75 ;

PUP DM S
(b) I = 25; 000 × 0:05 ×205
360 = PhP 711.81 ;

(c) I = 25; 000 × 0:05 ×209


360 = PhP 725.69 .

Remarks:

(1) Banks make more money in loans by charging more profitable interest on the principal
amount borrowed than the interest they pay on savings/depository accounts except for
time deposits or certificate of deposits wherein a higher interest is paid. Can you check
why?

(2) The amount of interest per period accrued under simple interest is fixed or constant (i.e. it
is always a percentage of the principal amount or mathematically, P × r every period).
Note: Review how to convert interest rates expressed in percent (%) to decimal.

Definition 24: Accumulated and Present Value


In an investment transaction, the value of a principal plus the total interest accrued
during the term is termed as its accumulated value or future value and we shall denote it
by A. Thus,

A=P+I

where P is the principal amount and I is the interest earned. Suppose at some time in the
future, we would like to obtain a certain amount of money. We will discount interest from
the accumulated value to determine its present value or the amount to be invested to
earn the desired interest. Moreover, the present value of a certian amount at the
beginning of the term is the principal value (P = A − I).

All Rights Reserved. 2020 Abdul, Atienza, et. al.


Lesson 5 67

Theorem 2: Accumulated Value under Simple Interest

If a principal amount P is invested for a term t (t ≥ 0) earns simple interest at a rate r,


then its accumulated value at the end of the term is given by

A=P+I
=P+Prt

PUP DM S
A = P(1 + r t):

Investments that involve compound interest may play an important role in reaching some of
your long term financial goals such as investments. An investment is the use of money or
capital for income or profit. We can divide investments into two classes: fixed investments and
variable investments. In a fixed investment, the amount invested as principal may be
guaranteed and the interest is computed at a fixed rate. Guaranteed means that the exact
amount invested will be paid back together with any accumulated interest. Examples of a fixed
investment are savings accounts, money market deposit ac counts, and certificates of deposit.
Another fixed investment is a government savings bond. In a variable investment, neither the
principal nor the interest is guaranteed. Examples of variable investments are stocks, mutual
funds, and commercial bonds.
Definition 25: Compound Interest

Investments made at a compound interest has the property that the interest earned at the
end of one period is automatically invested in the next period to earn additional interest.

Let us see how the two differ. Consider an amount of PhP of 1,000 which is invested at a
periodic interest rate of 5%. The amounts in the account after 3 periods assuming (a) simple
interest and (b) compound interest will be

(a) Simple Interest

Year 1 : 1; 000 + 0:05(1; 000) = 1; 050


Year 2 : 1; 000 + 0:05(1; 000) + 0:05(1; 000) = 1; 000(1 + 0:05(2)) = 1; 100
Year 3 : 1; 000 + 0:05(1; 000) + 0:05(1; 000) + 0:05(1; 000) = 1; 000(1 + 0:05(3)) = 1; 150
Indeed, from ??, the accumulated value of this principal amount at time t(t ≥ 0) will be A =
P(1 + r t).

(b) Compound Interest

All Rights Reserved. 2020 Abdul, Atienza, et. al.


Lesson 5 68

Year 1 : 1; 000 + 0:05(1; 000) = 1; 050


Year 2 : 1; 050 + 0:05(1; 050)) = 1; 050(1 + 0:05) = 1; 102:50
Year 3 : 1; 102:50 + 0:05(1; 102:50) = 1; 102:50(1 + 0:05) = 1; 157:625 ≈ 1;
157:63 Eventually at some time t, we will yield an accumulated value of
{z }
= 1; 000(1:05)t:
A = 1; 000
(1:05)(1:05)· · ·(1:05) |
PUP DM S t times

Remarks:

• Note that since interest may be compounded more often than once in a year, then the
effective interest rate per compounding period is given by rnwhere r is the compound
interest rate and n is the the frequency of conversion.

• If n = 1, then r is just the effective annual interest rate or simply the effective rate.

• If n = 2, then we say that the interest is compounded semiannually. If n = 4, then it is


compounded quarterly. And if n = 12, then we say that the interest is compounded
monthly.

Theorem 3: Accumulated Value under compound Interest

If a principal amount P is invested for a term t (t ≥ 0) earns interest at an effective rate r


compounded n times in a year, then its accumulated value at the end of the term is given
by
„ „ r «
A=P 1+n
r «„ r «
1+n 1+n ···
| {z }
n times in t periods
„ r «nt
=P 1+n :
Thus, the present value of such an amount A is
r «
„ 1 + n −nt
P=A
point (or we

Most debts and investments earn compound compound interest


in •
A
say, "interest is compounded"). An
terest, where interest is earned on both the
investment with compound interest will be
original principal and the interest up to that
worth more every year and thus will earn
more interest. Hence, banks charge
compound interest for long-term
transactions. t
1
1 + r simple interest 1

All Rights Reserved. 2020 Abdul, Atienza, et. al.


Lesson 5 69

Example 40. Regine invested PhP 80,000 in a savings account with an interest of 1.8%
convertible (compounded) monthly. If she, does not make any deposit nor withdrawal into this
account, determine the amount in the account after 2 years.

Solution. By the previous theorem,


≈ PhP

1 +0:018 12 82,930.23 :
A = 80; 000
«12×2

PUP DM S
Example 41. Cyrus would like to invest enough money in a certificate of deposit (CD) now to
pay for his son?s college expenses. If he estimates that he will need PhP 1,500,000 in 6 years,
how much should he invest now in a CD that has a rate of 2.5% compounded quarterly?

Solution. By the previous theorem,


≈ PhP

1 +0:025 4 129,166.48 :
P = 1; 500; 000
«4×6

Caution: Convert percentages to decimal carefully. Round off amounts of money to the nearest
centavo. Do not round off in the middle of the computation beacuse the final answer may be off
by a huge amount especially when dealing with large amounts of investments. Have a good
scientific calculator available to aid your computations.
5.2 Ordinary Annuities
Some payments are made regularly in more than one periods (monthly, quarterly or annually)
as in the case paying off a car loan.
Definition 26: Annuity

An annuity is an account into which, or out of which, a sequence of scheduled payments is


made.

There are many different types of annuities. An annuity may be an investment account that you
have with a bank, insurance company, or financial management firm. Annuities may contain
investments in stocks, bonds, mutual funds, money market accounts, and other types of
investments. Annuities are often used to save for long-term goals such as saving money for
college or for retirement. An annuity can also be used to provide long-term regular payments to
individuals. Some lottery jackpots and professional athletes’ salaries are paid out over time
from annuities. Retirees may invest some of their retirement savings (pension) into an annuity
and then receive monthly payments that come from that annuity rather than withdraw
everything as one lump sum.

All Rights Reserved. 2020 Abdul, Atienza, et. al.


Lesson 5 70

Consider an annuity with regular payouts of R at the end of each of the n periods such that it
earns interest at an effective rate of r per period. Then its present and accumulated values are
obtained as follows

P = R(1 + r)−1 + R(1 + r)−2 + · · · + R(1 + r)−n


(this is a geometric series with n terms and a common ratio (1 + r)−1)

PUP DM S
−n
= R ×1 − (1 + r)
1 − (1 + r)−1
−n
P = R ×1 − (1 + r)
r

and

A = R + R(1 + r) + +R(1 + r)2· · · + R(1 + r)n−1


(this is a geometric series with n terms and a common ratio (1 + r))
n
= R ×1 − (1 + r)
1 − (1 + r)
n
= R ×(1 + r) − 1
(1 + r) − 1
n
A = R ×(1 + r) − 1
r

Definition 27: Ordinary Annuity

An annuity into which equal payments are made at regular intervals, with the interest
compounded at the end of each interval and with a fixed interest rate for each
compounding period, is called an ordinary annuity or a fixed annuity.

Theorem 4: AV and PV of an Ordinary Annuity

If an ordinary annuity has regular payments of R at the end of each of the n periods and
earns interest at an effective rate of r per period. Then its present and accumulated
values are given by

−n
rand P = R ×1 − (1 + r)
n
r: A = R ×(1 + r) − 1

Remarks:

• Since annuities are mostly long-term transactions, we will assume that interest is
compounded at an effective rate of r every period

All Rights Reserved. 2020 Abdul, Atienza, et. al.


Lesson 5 71

• The n such regular payments noted above are said to be made at the end of each
payment period such that no additional payments nor withdrawals are made during the
term.

• All payments in the ordinary annuities considered in this material are considered
guaranteed pay ments. Such annuities are referred to as certain annuities. If payments
depend on the occurrence of an event then we call it a contingent annuity (e.g. payments
in an annuity from a pension fund are paid out as long as the annuitant is alive). The

latter is not covered in this material and

PUP

DMS
may be discussed in details in an advanced course in financial mathematics as it will
require deeper knowledge of probabilities.

Example 42. JB and Maryvhic are depositing PhP 10,000 each quarter in an ordinary annuity
that pays 4% interest compounded quarterly. Determine the accumulated amount in this
annuity after 5 years. How much total interest was earned?

Solution. Note that there are n = 4 × 5 = 20 quarterly payments and the quarterly rate is 1% so
the value of the annuity after 5 years is

20
A = 10; 000 ×(1:01) − 1

0:01= PhP 220,190.04 :

The total interest is hence equal to

I = Accumulated Amount − Total Payments


= 220; 190:04 − 10; 000(20)
= 220; 190:04 − 200; 000
I = PhP 20,190.04 :

Example 43. Jeremiah is willing to pay a loan with PhP 3,000 monthly installments for 3 years
at a rate of 8% convertible monthly. How much money can he borrow today?

Solution. Note that there are n = 12 × 3 = 36 monthly payments and the monthly rate is 8%/12 so
the
value of the loan 1− «36
is
1 +0:08 12 ≈= PhP 95,735.42
P = 3; 000 ×
0:08 :
„ 12

All Rights Reserved. 2020 Abdul, Atienza, et. al.


Lesson 5 72

5.3 Loan Repayment or Paying Off a Debt


A loan is an arrangement in which a lender gives money or property to a borrower, and the
borrower agrees to return the property or repay the money, usually along with interest, at some
future point(s) in time. Various methods of repaying a loan are possible. We will consider two of
them: The amortization method and the sinking fund method.
PUP DM S
Definition 28: Amortization Method
In the amortization method the borrower makes installment payments to the lender.
Usually these payments are at a regularly spaced periodic intervals; the progressive
reduction of the amount owed is described as the amortization of the loan. Examples
include car loan, home mortgage repayment.

When using the amortization method, the payments form an annuity whose present value is
equal to the original amount of the loan. In this section, we want to determine the unpaid
balance, also referred to as the outstanding loan balance or unpaid principal at any time after
the inception of the loan. There are two approaches used in finding the amount of the
outstanding balance: the prospective and the retrospective method which in general, are
equivalent.
(1) Pospective Method: the outstanding loan balance at any point in time is equal to the
present value at that date of the remaining payments.

(2) Retrospective Method: the outstanding loan balance at any point in time is equal to the
original amount of the loan accumulated to that date less the accumulated value at that
date of all payments previously made.
Theorem 5: Formula for Computing for the Outstanding Loan Balance
Let L be the principal amount borrowed which is to be repaid in n periods at an effective
interest rate of r per period. The prospective outstanding balance at the end of the kth
period is

Bk = R ×1 − (1 + r)−(n−k)
r

which is the present value at time k of the remaining n − k "future"


payments. The prospective outstanding balance at the end of the kth

period is

Bk = L(1 + r)k | {z } r
| {z }
value of the loan
accumulated value of all
at time k
k
"past" payments at time k
− R ×(1 + r) − 1
:

All Rights Reserved. 2020 Abdul, Atienza, et. al.


Lesson 5 73

Example 44. A loan is being repaid with level monthly payments of PhP 1,000. Calculate the
out standing balance of the loan if there are 12 payments left. The next payment will be paid
one year from now and the interest rate is 6% convertible monthly.
Solution. There are 12 monthly payments and we have no information about how much the
principal loan balance is and how many payments have been made previously so we compute
for the outstanding loan balance as follows

+0:06
1 «−12

PUP DM S
1− 0:06
B = 1; 000 × 12 ≈ PhP
11,618.93 :
12
Example 45. A loan of 200,000 is being repaid with quarterly payments of 20,000 at the end of
each year. The interest rate charged on the loan is 8% compounded quarterly. Calculate the
outstanding balance of the loan immediately after the 5th payment.

Solution. There were already 5 quarterly payments done and we have no information about
how many more payments are remaining so we compute for the outstanding loan balance
retrospectively as follows
« «5
1 +0:08 5 1 +0:08 − 1
„ „
4
B = 200; 000 − 20; 000 × ≈ PhP
116,735.36 :
4 0:08 4

When a loan is being repaid with the amortization method, each payment is partially a
repayment of principal and partially a payment of interest. Determining the amount of each for a
payment can be im portant (for tax purposes, for example). An amortization schedule is a table
which shows the division of each payment into principal and interest, together with the
outstanding loan balance after each payment is made.

Let L be the principal loan balance (i.e. the outstanding loan balance at time 0) being repaid
with regular installment payments of R, then an amortization schedule would look the table
below.

All Rights Reserved. 2020 Abdul, Atienza, et. al.


Lesson 5 74

Payment Interest Principal Outstanding Period Amount Paid Repaid Balance


.. .. .. .. ..
0 B0 = L 1 R I1 P1 B1 2 R I2 P2 B2 . . . . . n R In Pn Bn = 0
PUP DM S
Remarks:
−n
Total nR nR − R ×1 − (1 + r) r
!

−n
R ×1 − (1 + r) r

(1) The amortization method is the most common method of loan repayment. The
fundamental principle behind it is simple. When a payment is made, it must be first
applied to pay interest due and then any remaining part of the payment is applied to pay
principal.

(2) Fill up the table row by row from the top using the following:

• The regular installment payment is given by

R =L
r
1 − (1 + !
r)−n

where r is the effective rate of interest per period and n is the total number of
payment periods.

• On row/period t, the payment is a sum of interest paid and principal repaid (i.e. R = It
+Pt) • At period t, the interest paid is the interest rate effective per period r times the
previous balance or
It = r × Bt−1;

the principal repaid is


Pt = R − It;

while the outstanding balance is the former outstanding balance less the current
principal repaid (i.e. Bt = Bt−1 − Pt).

(3) The total principal repaid is the loan amount. The total interest paid is the difference
between the total payments made less the loan.

(4) The balance is fully paid off or is zeroed out by the end the loan’s term.

All Rights Reserved. 2020 Abdul, Atienza, et. al.


Lesson 5 75

Example 46. Consider a loan PhP 30,000 with level payments to be made at the end of every 2
months for 10 months at a bimonthly rate of 5% (need not divide by 6, this is already the
effective rate every 2-month period unless we say "compounded bimonthly"). Construct a loan
amortization schedule for this.

Solution. We first compute for the amount of each level payment. This is obtained as follows
!
≈ PhP 6,929.24 :
R =30; 000 1 − (1 +
0:05)−5

PUP DM S
0:05

Immediately fill the payment column with this amount.


Payment Interest Principal Outstanding
Period Amount Paid Repaid Balance
0 PhP 30,000.00
1 PhP 6,929.24 PhP 1,500.00 PhP 5,429.24 PhP 24,570.76
2 PhP 6,929.24 PhP 1,228.54 PhP 5,700.70 PhP 18,870.06
3 PhP 6,929.24 PhP 943.50 PhP 5,985.74 PhP 12,884.32
4 PhP 6,929.24 PhP 644.22 PhP 6,285.02 PhP 6,599.30
5 PhP 6,929.24 PhP 329.96 PhP 6,599.28 PhP 0.00

Remarks: Observe the following from the previous example.

• The last principal repaid is off by PhP 0.02 as an effect of rounding off the level payment
computed to the nearest centavo. As a rule of thumb, you may manually adjust this by
adding this to the last principal repaid. In practice, tables are created using spreadsheets
or computer programs to avoid this rounding off errors.

• The final balance is 0. The level payment fully pays off the loan as intended. • As

the balance decreases over time the amount of interest due in each period

decreases. • As the interest due goes down, the amount of principal paid in each

period increases.

• This method is typically applied to loans with level payments but the payments also may
change or vary over time as may apply to a payor who wish to make payment
adjustments in the middle of the term.

Example 47. Consider the loan PhP 30,000 in the previous example. Suppose the debtor can
only pay PhP 5,000 for each of the first 2 periods and catch up with a higher payment for the
final three periods. Construct a loan amortization schedule for this.

Solution. We first fill the first two payments with PhP 5,000.

All Rights Reserved. 2020 Abdul, Atienza, et. al.


Lesson 5 76

Payment Interest Principal Outstanding


Period Amount Paid Repaid Balance
0 PhP 30,000.00
1 PhP 5,000.00 PhP 1,500.00 PhP 3,500.00 PhP 26,500.00
2 PhP 5,000.00 PhP 1,325.00 PhP 3,675.00 PhP 22,825.00
3
Thus, the outstanding balance after two payments is PhP 22,825.00 which will be repaid with
higher

PUP

DMS
revised payments determined as follows
0:05)−3 0:05
R =22; 825 1 − (1 + !≈ PhP 8,381.54 :

As expected, it is higher than the first two payments.


Payment Interest Principal Outstanding
Period Amount Paid Repaid Balance
0 PhP 30,000.00
1 PhP 5,000.00 PhP 1,500.00 PhP 3,500.00 PhP 26,500.00
2 PhP 5,000.00 PhP 1,325.00 PhP 3,675.00 PhP 22,825.00
3 PhP 8,381.54 PhP 1,141.25 PhP 7,240.29 PhP 15,584.71
4 PhP 8,381.54 PhP 779.24 PhP 7,602.30 PhP 7,982.41
5 PhP 8,381.54 PhP 399.12 PhP 7,982.41 PhP 0.00

Note that we adjusted the last principal repaid manually by PhP 0.01 so the final balance

will be 0. Definition 29: Sinking Fund

An alternative for repaying a loan in installments by the amortization method, a borrower


can accumulate a fund which will exactly repay the loan in one lump sum at the end of a
specified period of time. This fund is called a sinking fund. It is generally required that the
borrower periodically pay interest on the loan, sometimes referred to as a service.

In this section, we shall only consider a sinking fund with fixed regular contributions. Because
the balance in the sinking fund could be applied against the loan at any point, the net amount
of the loan is equal to the original amount of the loan minus the sinking fund balance. It is
known that if the rate of interest paid on the loan equals the rate of interest earned on the
sinking fund, then the amortization method and the sinking fund method are equivalent.

Let L be the loan amount payable via sinking fund deposits D earning interest at an effective
rate k and interest due to the lender at an effective rate r per period. A sinking fund schedule
should look like the one below.

All Rights Reserved. 2020 Abdul, Atienza, et. al.


Lesson 5 77

Interest Sinking Fund Interest Amount Net Amount Period Payment Deposit
on the SF in the SF of the loan 0 SB1 = 0 B0 = L 1 Lr D I1 SB1 B1 2 Lr D I2 SB2 B2
.. .. .. .. .. ..
. . . . . . n Lr D In SBn = L Bn = 0

PUP

DMS
Remarks:

n
(1) If L is the loan amount then L = D ×(1 + k) − 1

kwhere n is the term of the loan and k is the


effective rate of interest per payment period on the sinking fund.

(2) Fill up the table row by row from top using the following:

• The interest payable to the lender each period is L × r.

• The sinking fund deposit is given by

D =L
k
(1 + k)n − !:
1
• The interest earned on the sinking fund balance during period t is It = k × SBt−1 where
k is charged to the previous sinking fund balance.

• At time t, the balance on the sinking fund is

SBt = SBt−1 + D + It:

• At time t the net amount of the loan or outstanding balance is the loan amount less
the current amount saved in the sinking fund or

Bt = L − SBt:

Example 48. Consider a loan PhP 30,000 to be paid with level interest due to the lender at the
end of every 2 months for 10 months at a bimonthly rate of 5% (need not divide by 6, this is
already the effective rate every 2-month period unless we say "compounded bimonthly") along
with deposits to a sinking fund that earns a bimonthly rate of 4%. Construct a loan sinking fund
schedule for this.

Solution. The interest due to the lender each period is L × r = 30; 000 × 0:05 = PhP 1; 500. We

then All Rights Reserved. 2020 Abdul, Atienza, et. al.

You might also like